You are on page 1of 88

Acknowledgements and Important Notice:

3B All questions from the HKDSE and HKCEE are reproduced by


permission of the HKEAA. Unauthorized use of the aforementioned
questions in this electronic version is prohibited.
Measures, Shape and Space

10 Mensuration (III)

10.0 Pre-Learning (P. 10.2)


10.1 Heights of Solids (P. 10.3)
10.2 Pyramids (P. 10.5)
10.3 Circular Cones (P. 10.16)
10.4 Spheres (P. 10.28)
10.5 Relations among Lengths,
Areas and Volumes of
Similar Figures (P. 10.38)
10.6 Frustums of Pyramids
and Circular Cones
(P. 10.51)

Zooming in and out ST EM


When using smartphones or tablets, we often drag the screen with our fingers to enlarge or reduce the
image of an object. The new image and the original image are similar figures. What is the relationship
between the areas of the new image and the original image?

Q&A In the figure, the radius of a circle on


1 cm
the screen of a smartphone is enlarged to
3 cm
3 times its original.
(a) Find the areas of the small and large
circles in terms of r.
(b) How many times is the area of the M
E
f

T
9 ith o

large circle that of the small circle?


.7 w ion
S
s t
P. ath lica
M pp
A

10
10.2 Chapter 10

10.0 Pre -Learning


Key Points
A. Circles and Sectors

Circle Sector
arc

diameter = d i • Arc length = i # 2rr


O • Circumference = rd or 2rr O 360c
radius = r
radius = r i # rr2
• Area = rr2 • Area of a sector =
360c

B. Prisms and Cylinders

Prism Cylinder

radius = r
base base
• Volume of cylinder
height = h = rr2h
curved height = h
• Curved surface area
surface
base • Volume of prism = A # h of right cylinder
• Total surface area of right base = 2rrh
Base area = A prism • Total surface area
Perimeter of base = P
= 2A + P # h of right cylinder
= 2rrh + 2rr2

C. Similar Plane Figures


Two figures having the same shape are called similar figures. In similar figures, all their
corresponding lengths are proportional.
e.g. (i) ABCD + PQRS (ii) 3ADE + 3ABC
A
P
B
D
S
D

B Q
A C
C R E

AB = BC = CD = DA AD = DE = AE
PQ QR RS SP AB BC AC

Warm-up Exercise
[In this exercise, give the answers in terms of r if necessary.]
1. Find the circumference and the area of a circle of radius 6 m.
Mensuration (III) 10.3

2. The figure shows a sector. Find


! A
(a) AB , 120° B
15 mm
(b) the area of sector OAB. O

3. The figure shows a right prism. Find 5 cm


12 cm
(a) the volume of the prism,
13 cm
8 cm
(b) the total surface area of the prism.

4. The figure shows a right cylinder. Find 2 cm


(a) the volume of the cylinder,
(b) the curved surface area of the cylinder, 7 cm
(c) the total surface area of the cylinder.

5. In the figure, ABCD a EFGH. Find the value of x. B


A E
F
9m xm

D G H
C 12 m 10 m

10.1 Heights of Solids


We learnt how to find the volumes and surface areas of prisms and
cylinders in S1 and S2. All these solids have uniform cross sections. In
this chapter, we are going to consider some solids without uniform cross
sections. However, before studying the mensuration of these solids, we need
to know the definition of a line perpendicular to a plane and the condition
for it, and hence learn more about the concept of the height of a solid.

A Projection of a Point on a Plane


In the figure, A is a point outside the plane and B is a point on the plane.
If the line AB is perpendicular to the plane, then AB is perpendicular to
every line on the plane which passes through B. In fact:
(i) B is the projection of A on the plane.
(ii) The length of AB is the perpendicular distance between A and
the plane.
A

m1 ◀ AB = l1,
B m3
m2 AB = l2,
AB = l3,
AB is perpendicular to the plane. ……

projection 投影
10.4 Chapter 10

On the contrary, if the line AB is perpendicular to two lines on a plane Quick Quiz
which pass through B, then we can determine that AB is perpendicular to
P
the plane.
A
S

Q
◀ AB = l1,
m1
AB = l2. R
B m2
In the figure, is the
projection of on the
B Concept of the Height of a Solid plane.

In Book 1B Chapter 7, we learnt that the polygons at both ends of a prism


are called the bases, and the perpendicular distance between the two
bases is called the height of the prism.

base

height
base

In fact, any face of a prism has its corresponding height.


Fig. 1(a) shows a right prism. Its base is a right-angled triangle. We can C

see that: B

1. C is the projection of A on the face BCDF, i.e. the length of AC is A

the perpendicular distance between A and the face BCDF. D

2. Among the perpendicular distance between each point on the faces of F

the prism and the face BCDF, AC is the longest. E


Fig. 1(a)
For the face BCDF, we can say that its corresponding height is AC.
Similarly, for the same face BCDF, ED is another corresponding height.
Now, let’s look at Fig. 1(b) and Fig. 1(c) which show the corresponding
heights of two other selected faces of the prism in Fig. 1(a).
From Fig. 1(a)–(c), we see
For the face AEDC: For the face ABFE: that any face of a prism has
Corresponding height Corresponding height its corresponding height.
C BC (or FD) C CG (or DH)

B B
G
A A

D D

F F
H
E E
Fig. 1(b) Fig. 1(c)
Mensuration (III) 10.5

In fact, the above-mentioned concept involving heights not only applies to


prisms, but also applies to other solids such as circular cylinder, pyramids
and circular cones. Let’s look at the following examples.

A P X

B Q Y

The corresponding height The corresponding height The corresponding height


of the shaded face is AB. of the shaded face is PQ. of the shaded face is XY.

We will further discuss pyramids and circular cones in the following sections.

10.2 Pyramids
Pyramids are solids without uniform cross sections. Below are some daily
examples of pyramids.

Pyramid of Khufu Rice dumpling Tea bag

vertex
In general, if a solid has a polygonal base and all its other faces are V
triangles with a common vertex, then the solid is a pyramid (see slant edge

Fig. 2(a)), where


lateral face
1. the quadrilateral ABCD is called the base, D C
2. the triangular faces (such as VBC) are called lateral faces,
A
3. all the lateral faces meet at a point V, which is called the vertex, base B

4. the common edge of any two adjacent lateral faces (such as VC) is Fig. 2(a)

called a slant edge.


V
Refer to Fig. 2(b). height of
the pyramid
(i) E is the projection of V on the base ABCD.
(ii) Among the perpendicular distance between each point on the faces
D
of the pyramid and the base ABCD, VE is the longest. C

Hence, for the base ABCD, VE is the corresponding height. E


A B
In this case, the perpendicular distance VE between the vertex and the
base is called the height of the pyramid. Fig. 2(b)

pyramid 角錐 slant edge 斜稜


10.6 Chapter 10

A pyramid can be named according to the shape of its base.


e.g.

triangular pyramid rectangular pyramid pentagonal pyramid

Fig. 3 shows two pyramids with rectangular bases. Let N be the point of
intersection of the diagonals of the base.
If N is the projection of the vertex V on the base (i.e. VN is the height),
then the pyramid is called a right pyramid [Fig. 3(a)].
If N is not the projection of the vertex V on the base (i.e. VN is not the
height), then the pyramid is not a right pyramid and it is called an oblique
pyramid [Fig. 3(b)]. More to Learn
V V In general, the vertex of a
right pyramid is vertically
above the centroid of its
D
C base.
e.g.
N N
A
B

(a) Right pyramid (b) Oblique pyramid


Fig. 3

The right pyramid VABCD in Fig. 3(a) has the following properties:
centroid of base
1. The foot of perpendicular from the vertex of the pyramid to the base is
equidistant from each vertex of the base.
i.e. NA = NB = NC = ND.
2. The slant edges are of equal length.
i.e. VA = VB = VC = VD.
3. All the lateral faces are isosceles triangles. Think
i.e. 3VAB, 3VBC, 3VCD and 3VDA are isosceles triangles. 1. Are the lateral faces
of a right pyramid
If the base of a right pyramid is a regular polygon, this pyramid is called
congruent?
a regular pyramid. The three properties mentioned above are also true
2. Are the lateral faces
for all regular pyramids.
of a regular pyramid
congruent?

right pyramid 直立角錐 oblique pyramid 斜角錐 regular pyramid 正角錐


Mensuration (III) 10.7

A Volume of a Pyramid

Class Activity 10.1 Class e-Activity

Fig. A shows a cube of side a. The cube can be divided into three identical pyramids, as shown in Fig. B.

1 #*3
a

a
a
x
Fig. A Fig. B
3
1. Consider Fig. A. Volume of the cube = a

2. Consider Fig. A and Fig. B. Volume of each pyramid = 5 # volume of the cube

3. The figure on the right shows one of the pyramids. asox


height
a
(a) Base area of the pyramid = a
a
(b) Height of the pyramid = a

4. Hence,
volume of each pyramid
= # volume of the cube
= # a3
= # a2 # a
= # of each pyramid # of each pyramid

In general, for any pyramid,

volume of a pyramid = 31 # base area # height

Quick Quick Quiz


Example Volume of the triangular pyramid in the figure
8m
= 31 # 15 # 8 m3 Find the volume of the
pyramid below.
= 40 m3
2
base area = 15 m height = 12 cm

2
base area = 160 cm
10.8 Chapter 10

Example 1 In the figure, VABCD is a rectangular pyramid with a V

Level 1 square base. Its volume is 108 cm3 and its height is 9 cm. 9 cm

Find the length of a side of its base.

Solution Let l cm be the length of a side of its base.


D C
1 2 N l
3 # l # 9 = 108 A l B
2
l = 36
l=6
` The length of a side of its base is 6 cm.

Instant Drill 1
The figure shows a pyramid with a rectangular
base of length 8 cm and width 7 cm. Its volume
is 168 cm3. Find the height of the pyramid.
7 cm
8 cm
➥ Ex 10A 7, 10, 11

Example 2 In the figure, VABCD is a right pyramid. Its base is a V

Level 2 rectangle of length 12 cm and width 9 cm. If the length of


10 cm
the slant edge VC is 10 cm, find
D
(a) the height VN of the pyramid, C
A N
(b) the volume of the pyramid. 12 cm
9 cm
B
(Give the answers correct to 3 significant figures.)

Solution (a) In the right-angled triangle ABC, A

AC = AB2 + BC2 (Pyth. theorem)


9 cm
= 92 + 122 cm
= 15 cm B C
12 cm

` NC = 21 # AC

= 21 # 15 cm
= 7.5 cm
In the right-angled triangle VNC, V

VN = VC2 - NC2 (Pyth. theorem) 10 cm

= 102 - 7.52 cm N C
7.5 cm
= 6.61 cm, cor. to 3 sig. fig. 6.614 4
` The height of the pyramid is 6.61 cm.
Mensuration (III) 10.9

(b) Volume of the pyramid = 31 # (12 # 9) # 6.614 4 cm


3

= 238 cm3 , cor. to 3 sig. fig.

Instant Drill 2
The figure shows a right pyramid with a square 8 cm

base. Find
5 cm
(a) the height of the pyramid,
(b) the volume of the pyramid.
(Give the answers correct to 2 significant figures.)
➥ Ex 10A 15, 17

B Total Surface Area of a Right Pyramid


The figure below shows a net of the right pyramid VABCD obtained by
cutting along its slant edges.
V

lateral lateral lateral


V slant edges D C
face face face

V base V
D
C
A B A lateral B
face

From the figure above, the surface of the pyramid consists of a base and
all lateral faces. Hence,

total surface area of a right pyramid


= total area of all lateral faces + area of the base

Quick
Example For the right pyramid in the figure, Quick Quiz
V
total area of all lateral faces area of each
Find the total surface area
= 4 # 30 cm2 lateral face
of the right pyramid below.
= 30 cm2
= 120 cm2 6 cm

total surface area 6 cm

= (120 + 5 # 5) cm2 D C

= 145 cm2 5 cm
A
5 cm B area of each
lateral face
2
= 45 cm
10.10 Chapter 10

Example 3 The figure shows a tent in the shape of a regular pyramid. V

Level 1 The four lateral faces and the base are made of the same
material. A side of the base is 3 m long. The height of
the lateral face 3VBC is 4 m. Find the total area of the D
C
material used to make the tent. A
3m E
B 4m

Solution Area of 3VBC = 21 # 3 # 4 m2


= 6 m2
Total area of the material used
= 4 # area of 3VBC + area of square ABCD ◀ a The base of a regular
= (4 # 6 + 32) m2 pyramid is a regular polygon.
` 3VAB, 3VBC, 3VCD
= 33 m2 and 3VDA are congruent
triangles (SSS).
Instant Drill 3
Find the total surface area of the regular
pyramid in the figure. 5 cm

6 cm

➥ Ex 10A 4–6

Example 4 In the figure, VABCD is a right pyramid whose base is a V

Level 2 rectangle of length 30 cm and width 14 cm. The length of 25 cm


the slant edge VB is 25 cm.
D C
(a) Find the area of 3VAB. A 14 cm
30 cm B
(b) Find the area of 3VBC.

(c) Hence, find the total surface area of the pyramid.

Solution (a) Let VM be the height of 3VAB. V

MB = 21 # AB ◀ a 3VAM , 3VBM (RHS) 25 cm


` MA = MB
= 21 # 30 cm A B
M
30 cm
= 15 cm
In 3VMB,
VM = VB2 - MB2 (Pyth. theorem)
2 2
= 25 - 15 cm
= 20 cm
Mensuration (III) 10.11

` Area of 3VAB = 21 # AB # VM

= 21 # 30 # 20 cm2
= 300 cm2

(b) Let VN be the height of 3VBC. V

NB = 21 # BC ◀ a 3VBN , 3VCN (RHS) 25 cm


` NB = NC
= 21 # 14 cm
B C
N
= 7 cm 14 cm

In 3VNB,
VN = VB2 - NB2 (Pyth. theorem)

= 252 - 72 cm
= 24 cm
` Area of 3VBC = 21 # BC # VN

= 21 # 14 # 24 cm2
= 168 cm2

(c) Total surface area of the pyramid


= 2 # area of 3VAB + 2 # area of 3VBC + ◀ 3VAB , 3VCD (SSS)
area of rectangle ABCD 3VBC , 3VDA (SSS)

= (2 # 300 + 2 # 168 + 30 # 14) cm2


= 1 356 cm2

Instant Drill 4
The figure shows a right triangular pyramid.
The length of each edge is 10 cm. Find 10 cm

(a) the area of each lateral face,


(b) the total surface area of the pyramid. ➥ Ex 10A 19–21
10 cm
(Give the answers correct to 3 significant figures.) Public Exam 8
Question
10.12 Chapter 10

Class Practice 10.1


Find the volume of each of the following pyramids. [Nos. 1–3]
1. V 2. 3. V

V
7m 12 cm
F E
10 cm D C
A D C
6c
m X
cm
B C 5 O
base area = 90 m
2
A B A 10 cm B
ABCD is a square.

Find the total surface area of each of the following right pyramids. [Nos. 4–6]
V V
4. 5. 6.
8m

V
8 cm 17 cm
D D C
D C
C

M N
6m
6 cm A M
A B A B
B 15 cm
ABCD is a square. ABCD is a square. ABCD is a square.

7. The figure shows a pyramid whose base is a rectangle of length 16 cm


and width 14 cm. If the volume of the pyramid is 896 cm3, find its
height.

16 cm

14 cm

8. In the figure, the base of the pyramid is a right-angled triangle ABC, V

where AB = BC. If the volume of the pyramid is 80 cm3, the height


VA = 10 cm and AB = 8 cm, find
10 cm
C
(a) the area of 3ABC,

(b) the length of BC.


A B
8 cm
(c) the length of AC.
Mensuration (III) 10.13

m ulae
For
Exercise 10A
• Volume of a pyramid
[In this exercise, give the answers correct to 3 significant figures
= 1 # base area # height
3
if necessary.]
• Total surface area of a right pyramid
= total area of all lateral faces + area
Level 1 of the base

Find the volume of each of the following pyramids. [Nos. 1–3]


1. 2. 3.

8 mm
10 m
height = 11 m

4m
5m 6m

base area = 33 mm2 12 m 10 m


The base is a parallelogram. The base is a trapezium.

Find the total surface area of each of the following right pyramids. [Nos. 4–6]
4. 5. 6. V
9 cm
area of each 7 cm
lateral face
= 40 mm2 18 cm D C

m
15 cm F

4c
A E B
7 mm
7 mm 15 cm 12 cm
➥ Example 3

Find the unknown in each of the following right pyramids. [Nos. 7–9]
7. 8. 9.
area of each y cm
lateral face
= 30 m2
h cm

5 cm xm 9 cm

6 cm ➥ Example 1 xm 9 cm
volume = 70 cm3 total surface area = 156 m2 total surface area = 225 cm2

3
10. In Egypt, there is a right pyramid with volume 2 126 350 m . The
base of the pyramid is a square of side 215 m. What is the height of
the pyramid?
10.14 Chapter 10

11. The figure shows a right pyramid whose base ABCD is a rectangle. V

If the volume of the pyramid is 2 880 cm3, BC = 40 cm and height


12 cm
VN = 12 cm, find A D

(a) the area of the base ABCD, N

B 40 cm C
(b) the length of CD.

12. In the figure, the base of the pyramid VABC is a right-angled triangle V

ABC, where AB = 16 cm, AC = 12 cm and +BAC = 90c. If its


volume is 640 cm3 and its height VA = h cm, find h cm

(a) the value of h, 16 cm


B A
(b) the length of the slant edge VB. 12 cm

13. The base of the pyramid in the figure is a right-angled triangle XYZ, V

where XZ = YZ, YZ = 15 cm and XY = 17 cm. The height VZ of the


pyramid is 18 cm. Find
18 cm
Y 17 cm
X
(a) the length of XZ,
15 cm
(b) the volume of the pyramid.
Z

14. The cardboard in the figure is the net of a regular pyramid. ABCD is V

a square of side 20 cm. It is given that VC = 26 cm. Find 26 cm

D C
(a) the length of VE, E
V V
(b) the total surface area of the pyramid.
A B 20 cm

V
Level 2

15. In the figure, the base of the pyramid is a right-angled triangle ABC, V

where AC = BC. VC is the height of the pyramid. If AC = 16 cm, 20 cm


13 cm
VB = 13 cm and VA = 20 cm, find
16 cm
(a) the height of the pyramid, C A

B
(b) the length of BC,

(c) the volume of the pyramid. ➥ Example 2

16. The base of a solid metal right pyramid is a rectangle with


Explain
dimensions 16 cm by 9 cm. The pyramid is now melted and recast
into a solid regular pyramid with a square base of side 12 cm. Jacky
claims that the height of the new pyramid is larger than that of the
original one. Do you agree? Explain your answer.
Mensuration (III) 10.15

17. The figure shows a right pyramid with a rectangular base PQRS. The V

diagonals of the base intersect at M. If PQ = 24 cm, QR = 18 cm 25 cm


and VR = 25 cm, find
S R
(a) the length of MR,
M 18 cm
(b) the height VM of the pyramid, P 24 cm Q
(c) the volume of the pyramid.

18. The figure shows a pyramid whose base is a rhombus. The diagonals V

of the base intersect at N and AC = 8 cm. If the height VN of the


pyramid is 5 cm and its volume is 40 cm3, find D C

(a) the area of the base ABCD, N


A
(b) the length of BD. B

19. The figure shows a model of a pavilion whose roof is in the shape 20 cm

of a right pyramid. The base of the roof is a square of side 24 cm


and the length of each slant edge is 20 cm. Find the total area of all 24 cm

lateral faces of the roof of the model. ➥ Example 4

20. In the figure, the base ABCD of a right pyramid is a rectangle with V
8 cm
dimensions 30 cm by 12 cm. The height VP of the pyramid is 8 cm.
Find D C

(a) the lengths of VM and VN, N

cm
P

12
(b) the total surface area of the pyramid. A B
M
30 cm

21. In the figure, VABCD is a right pyramid. Its base ABCD is a V

rectangle. If AB = 10 cm, BC = 8 cm and VH = 12 cm, find 12 cm

(a) the area of 3VAB,


D
(b) the length of VK and hence the area of 3VBC,
A
(c) the total surface area of the pyramid. C
10 H
cm K
B
8 cm

22. The figure shows a regular pyramid VABCD. Its base is a square of V
C
side 16 cm. The total surface area of the pyramid is 576 cm2. Find D

(a) the area of 3VAB, O


E

(b) the height VO of the pyramid, A


B
16 cm

(c) the volume of the pyramid.


10.16 Chapter 10

Level 3
T T T
23.

P P
S S
U U

Q R Q R Q R

Fig. A Fig. B

In Fig. A, PQRSTU is a wooden block in the shape of a right prism,


where PQ = PR, QR = 12 cm and PT = 24 cm. The wooden block
PQRSTU is now cut along plane QRT, and two wooden blocks PQRT
and TQRSU are formed as shown in Fig. B. If the volume of wooden
block PQRT is 384 cm3, find
(a) the area of 3PQR,

(b) the length of RT,

(c) the area of 3QRT.

24. The figure shows a regular tetrahedron VABC which is formed by V

four equilateral triangles. The length of each edge is 6 cm. CD is a A

median of 3ABC. It is given that DO : OC = 1 : 2. Find


D C
(a) the length of CD, O

(b) the total surface area of the regular tetrahedron,


B
(c) the height VO of the regular tetrahedron,
(d) the volume of the regular tetrahedron.
(Leave the radical sign ‘ ’in the answers.)

10.3 Circular Cones


The following show some common objects. Their shapes are known as
circular cones.

Traffic cone Ice cream cone Party hat

circular cone 圓錐
Mensuration (III) 10.17

A circular cone is a solid enclosed by a circular plane and a curved vertex


V
surface (see Fig. 4(a)). For a circular cone,
1. the circular plane is called the base, slant height

2. the top is a point V called the vertex,


base Y
3. the distance between the vertex and any point on the circumference of
the base (e.g. VY) is called the slant height. Fig. 4(a)

Refer to Fig. 4(b). V


(i) X is the projection of V on the base.
height of
(ii) Among the perpendicular distance between each point on the curved the circular
cone
surface of the circular cone and the base, VX is the longest.
base Y
Hence, for the base in Fig. 4(b), VX is the corresponding height. X

In this case, the perpendicular distance VX between the vertex and the Fig. 4(b)

base is called the height of the circular cone.

Fig. 5 shows two circular cones. Let O be the centre of the base.

If O is the projection of the vertex V on the base (i.e. VO is the height),


then the cone is called a right circular cone [Fig. 5(a)].
If O is not the projection of the vertex V on the base (i.e. VO is not the
height), then the cone is not a right circular cone and it is an oblique
circular cone [Fig. 5(b)]. -

V V

O O

(a) Right circular cone (b) Oblique circular cone


Fig. 5

In a right circular cone, all slant heights are equal.


Let l be the slant height, r be the base radius and h be the height of a V

right circular cone. By Pythagoras’ theorem, we have


m h
2 2 2
l =r +h
A O
r

slant height 斜高 right circular cone 直立圓錐 oblique circular cone 斜圓錐
10.18 Chapter 10

A Volume of a Circular Cone


Consider a pyramid with a regular polygon as its base. When the number
of sides of the base of the pyramid increases indefinitely, the base will
tend to a circle. Hence, the pyramid will tend to a circular cone.

Since a circular cone can be obtained from a pyramid through the above
changes, by using the formula of the volume of a pyramid, we have
volume of a circular cone = 31 # base area # height

Therefore, if the base radius of a circular cone is r and the height is h,


then

volume of a circular cone = 31 rr2h ◀ Recall:


Area of a circle = rr2
h h

r r
Right circular cone Oblique circular cone

Quick Quick Quiz


Example Volume of the circular cone in the figure

= 31 # r # 42 # 9 cm3 height = 9 m
Find the volume of the right
circular cone below in terms
= 48r cm3 of r.
4 cm
5 cm

6 cm

Example 5 The figure shows an ice cream cone in the shape of a 5 cm

Level 1 right circular cone. Its base diameter is 5 cm and its slant
height is 12 cm. Find the volume of the ice cream cone,
correct to the nearest cm3. 12 cm
Mensuration (III) 10.19

Solution Let h cm be the height of the ice cream cone. 5


2
cm
2 Clue
a 5 k + h2 = 122 (Pyth. theorem)
2 First find the height of
2 2
h = 12 - 2.5 the ice cream cone by
12 cm h cm Pythagoras’ theorem.
= 137.75
` Volume of the ice cream cone
Calculator
= 31 # r # 2.52 # 137.75 cm3 Keying sequence:
3 3 1 ÷ 3 × SHIFT EXP ×
= 77 cm , cor. to the nearest cm
2.5 x 2 × √–– 137.75 EXE
[76.81…]
Instant Drill 5
Find the volume of the right circular cone 16 cm
in the figure in terms of r. 20 cm

➥ Ex 10B 3

Example 6 In the figure, the base radius of a solid circular chocolate


Level 2 cone is 9 mm and the height is 8 mm. It is melted to form
Cross-topic a solid chocolate cylinder of height 6 mm. Find the base
radius of the cylinder.

8 mm 6 mm

9 mm

Solution Let r mm be the base radius of the cylinder.


Volume of the circular cone = volume of the cylinder
1 2 2
3 # r # 9 # 8 = rr # 6
◀ Recall:
Volume of a cylinder = rr2h
36 = r2
r=6
` The base radius of the cylinder is 6 mm.

Instant Drill 6
In the figure, the base radius of the solid metal cylinder is 4 cm and
the height is 12 cm. It is melted and recast into a solid right circular
cone of base radius 8 cm. Find the height of the circular cone.
12 cm
4 cm

8 cm
➥ Ex 10B 15
10.20 Chapter 10

B Total Surface Area of a Right Circular Cone

Class Activity 10.2 Class e-Activity

Kate cuts out the sector OAB in Fig. A and rolls it to make radii OA and
OB coincide. She uses a tape to fix it and obtains the right circular cone
as shown in Fig. B. A B

1. (a) Consider Fig. A. AB = i # 2rl


!
m i
360c
Consider Fig. B. O

Circumference of the base of the circular cone = Fig. A

(b) Consider Fig. A and Fig. B. What is the relationship between


!
AB and the circumference of the base of the circular cone?
O
Ans

(c) Using the results above, we have m


i # 2rl =
360c
r
i = ......................................... (1)
360c A, B
Fig. B

2. (a) Consider Fig. A again.

Area of sector OAB = i # ............... (2)


360c
(b) Substitute (1) into (2).

Area of sector OAB = #


=

3. a Area of sector OAB in Fig. A = curved surface area of the circular cone in Fig. B

` Curved surface area of the circular cone in Fig. B =

From Class Activity 10.2, for a right circular cone of base radius r and
slant height l,

m
curved surface area of a right circular cone = rrl

r
Mensuration (III) 10.21

When the area of the base of a right circular cone is added, we have
total surface area of a right circular cone
= curved surface area + area of base
Therefore, for a right circular cone of base radius r and slant height l,

total surface area of a right circular cone = rrl + rr2

Quick Quick Quiz


Example For the right circular cone in the figure,
2
curved surface area = r # 3 # 7 cm Find the total surface area
7 cm
= 21r cm2 of the right circular cone
below in terms of r .
total surface area = (21r + r # 32) cm2 3 cm

= 30r cm2
5 cm

4 cm

Example 7 In the figure, the base radius of a right circular cone is V

Level 1 6 cm and the total surface area is 96r cm2. Find the slant
height VQ of the circular cone.

Solution Let l cm be the slant height VQ. P O 6 cm


Q

r # 6 # l + r # 62 = 96r
6rl + 36r = 96r
6rl = 60r
l = 10
` The slant height VQ of the circular cone is 10 cm.

Instant Drill 7
In the figure, the base radius of a right V

circular cone is 8 cm and the total surface


area is 200r cm2. Find the slant height VB of
the circular cone.
A O 8 cm B

➥ Ex 10B 8
10.22 Chapter 10

Example 8 In the figure, a paper sector is folded into a vessel in the


Level 2 shape of a right circular cone. The radius of the sector is
Cross-topic 15 cm and the angle of the sector is 288c.
Explain

288° A,B

O
O
15 cm

A B

(a) Find the base radius of the vessel.


3
(b) Are 100 vessels enough to hold 0.2 m of juice?
Explain your answer.

Solution (a) Let r cm be the base radius of the vessel. Diagram Clue
Radius of sector = slant height of vessel Consider the areas.

= 15 cm 288° r cm

Area of sector = curved surface area of vessel =

cm
15 cm

15
288c
# r # 152 = r # r # 15
360c
r = 12
` The base radius of the vessel is 12 cm.

Alternative
Let r cm be the base radius of the vessel. Diagram Clue
Length of the major circumference of the Consider the lengths of the curves.
=
arc AB in the sector base of the vessel
288° r cm
288c
# 2r # 15 = 2rr
360c =
cm

r = 12
15

` The base radius of the vessel is 12 cm.


Mensuration (III) 10.23

(b) Explain Tutor


• Find the height of the vessel.
• After finding the total capacity of the 100 vessels,
convert its unit into m3, then compare the result with
the datum in the question.
• Write down the conclusion.

Let h cm be the height of the vessel. 12 cm

h = 152 - 122 (Pyth. theorem)


h cm
15 cm
=9
Total capacity of 100 vessels
= 100 # 31 # r # 122 # 9 cm3
= 43 200r cm3
= 43 200r m3 ◀ 1 m = 100 cm
100 # 100 # 100
1 m3 = 100 # 100 # 100 cm3
= 0.136 m3, cor. to 3 sig. fig.
1 0.2 m3
` 100 vessels are not enough to hold 0.2 m3 of juice.

Instant Drill 8
The figure shows a sector OAB. If the sector is folded into B
A 10 cm O
a hollow right circular cone, find O

216°
(a) the base radius of the circular cone,
A,B
(b) the capacity of the circular cone in terms of r.

➥ Ex 10B 17, 18

Class Practice 10.2


[In this exercise, give the answers correct to 3 significant figures if necessary.]
Find the volume of each of the following circular cones. [Nos. 1–2]
1. 2. 9 cm

height = 11 m
15 cm

8m

hollow 空心的
10.24 Chapter 10

Find the total surface area of each of the following right circular cones. [Nos. 3–4]
3. 4.
12 cm
12 mm

10 cm
5 mm

5. In the figure, the base radius of the circular cone is 7 cm and its V

volume is 392r cm3. Find


(a) the height VO of the circular cone,

(b) the total surface area of the circular cone.


A O B
7 cm

6. In the figure, the base radius of an inverted right circular cone is A


21 cm
O B
21 cm and the total surface area is 1 050r cm2. Find
(a) the slant height VA of the circular cone,
V
(b) the volume of the circular cone.

m ulae
For
Exercise 10B
• Volume of a circular cone
[In this exercise, unless otherwise stated, give the answers correct to
= 1 rr2h
3 significant figures if necessary.] 3
• Curved surface area of a right
Level 1 circular cone = rrl
• Total surface area of a right
Find the volume of each of the following circular cones in circular cone = rrl + rr2
terms of r. [Nos. 1–3]
1. 2. 3.
26 cm
12 cm 17 cm

7 cm 18 cm 8 cm
➥ Example 5
Mensuration (III) 10.25

4. The figure shows a circular cone of base radius r and height h.


Complete the following table.
h
Base radius Height Volume
(a) 3m 24r m3 r
3
(b) 15 cm 80r cm

For each right circular cone in Nos. 5–7, find


(a) the curved surface area,

(b) the total surface area.


(Give the answers in terms of r.)
5. 6. 7.
28 cm 24 cm

20 cm
34 cm

16 cm 10 cm

8. The figure shows a right circular cone of base radius r and slant r
height l. Complete the following tables.
m
Base radius Slant height Curved surface area
(a) (i) 7 mm 35r mm2

(ii) 11 cm 132r cm2

Base radius Slant height Total surface area


(b) (i) 5m 70r m2

(ii) 8 cm 184r cm2 ➥ Example 7

9. Mr Chan pours 84 cm3 of juice into a glass. The shape of the glass is a
Explain
right circular cone with base radius 4 cm and height 5 cm. Will the
juice overflow? Explain your answer.
10.26 Chapter 10

10. The figure shows a tent of a Red Indian tribe in North America in
the shape of a right circular cone. Its base diameter is 4.8 m and its
slant height is 4 m. Find
(a) the height of the tent,

(b) the volume of the tent.

11. A birthday hat is in the shape of a right circular cone. Its base radius
is 7 cm and its height is 24 cm. Find
(a) the slant height of the hat,

(b) the curved surface area of the hat.

2
12. The base area of a right circular cone is 64r cm and its height is
15 cm. Find
(a) the base radius of the circular cone,

(b) the slant height of the circular cone,

(c) the total surface area of the circular cone in terms of r.

3
13. In the figure, the volume of a right circular cone is 324r cm and its
base radius is 9 cm. Find
(a) the height of the circular cone,
9 cm
(b) the slant height of the circular cone,

(c) the total surface area of the circular cone in terms of r.

14. It is known that the curved surface area of a right circular cone is
260r cm2 and its slant height is 26 cm. Find
(a) the base radius of the circular cone,

(b) the height of the circular cone,

(c) the volume of the circular cone in terms of r.

Level 2

15. In the figure, a cylindrical vessel of base radius 3 cm


contains a cocktail of depth 7 cm. If all the cocktail
8 cm
is poured into a glass in the shape of an inverted right
circular cone as shown, the glass is just filled up. Find the
7 cm
base radius of the glass. ➥ Example 6

3 cm
Mensuration (III) 10.27

16. The picture shows a Mongolian tent. Its shape is


3m
a right cylinder with a right circular cone on the
top. Its dimensions are shown in the figure. 3.2 m

(a) Find the volume of the tent.


4m
(b) The framework of the tent is made of timber
and the outer surface is covered with a
blanket. Find the total area of the blanket
required to build the tent.
(Give the answers in terms of r.)

17. Fig. A shows a paper cup in the shape of an inverted Q 4 cm

right circular cone. Its base radius is 4 cm and its


8.5 cm
slant height is 8.5 cm.
Q
(a) Find the capacity of the paper cup. P P
Fig. A Fig. B
(b) The paper cup is cut along PQ to form the sector
in Fig. B. Find
(i) the area of the sector,
(ii) the angle of the sector. ➥ Example 8

18. If a popcorn cup in the shape of a right circular cone


101° 25 cm
is cut along its slant height, a sector of radius 25 cm
and angle of sector 101c is obtained as shown. Find
(a) the base radius and the height of the cup,

(b) the capacity of the cup.

19. In the figure, a triangular board ABC is right-angled at B. The


lengths of the two legs are 21 cm and 28 cm. If it is rotated about A
21 cm
its hypotenuse, two circular cones are obtained.
N B
(a) Find the length of BN.
28 cm
(b) Find the total curved surface area of the two cones in terms of r.
C
(c) Find the total volume of the two cones in terms of r.
10.28 Chapter 10

20. The hourglass in the figure consists of two right circular cones with 3 cm

different heights. The base radii of both circular cones are 3 cm. If
the height of the hourglass is 10 cm, find its capacity.
10 cm

21. In the figure, the right cylinder and the right circular
cone have the same base radius. The height of the
circular cone is 2 times that of the cylinder. It is given
that the volume of the cylinder is 384r cm3. 2 h cm

(a) Find the volume of the circular cone in terms of r. h cm

(b) If the ratio of the base radius of the circular r cm


r cm
cone to its height is 2 : 3, find the height of the
circular cone.

Level 3

22. Two identical open right circular cones, A and B as shown, each
15 cm B 15 cm
A
with a base radius of 8 cm and a height of 15 cm, are both cut along
their slant heights. Then, the two sectors formed are joined and
folded to form a larger open right circular cone C. 8 cm
8 cm

(a) Find the radius of each of the two sectors obtained.

(b) Find the base radius and the height of the new cone C.
Explain (c) Which is larger, the volume of the space in C or the total C

volume of the space in A and B? Explain your answer.

10.4 Spheres
A sphere is a solid which appears everywhere in daily life. The following
are some examples:

Tennis ball Globe Collection of the


Vatican Museum

sphere 球體
Mensuration (III) 10.29

Fig. 6 shows a sphere. On the surface of a sphere, all points are


equidistant from a fixed point (O) inside the sphere. The fixed point is
called the centre. The distance between the centre and any point on the
surface is the radius (r).

centre

r ◀ The length of the diameter is


O
twice that of the radius, i.e. 2r.

Fig. 6

When a sphere is cut along a plane which passes through the centre, it is Interesting Maths
divided into two identical parts called hemispheres. Hong Kong Space Museum
looks like a hemisphere.

O r

O r

More to Learn
A hemisphere contains a plane which is in the shape of a circle. This
Archimedes (287 BC -212
circle is the base of the hemisphere and it has the same centre and radius
BC) discovered that the
as the original sphere. volume of a sphere is 2
3
times the volume of its
circumscribing cylinder.
A Volume of a Sphere

If the radius of a sphere is r, then r


2r

volume of a sphere = 43 rr3

r
Using this result, we can
easily deduce the formula of
the volume of a sphere.

Quick Quiz
Quick Find the volume of the
Example Volume of the basketball in the figure sphere below, correct to the
= 43 # r # 123 cm3 nearest m3.

= 2 304r cm3 5m

12 cm

centre 球心 hemisphere 半球體


10.30 Chapter 10

Example 9 The figure shows a solid ice cream model which consists
Level 1 of a hemisphere and an inverted right circular cone with 6 cm
a common base. The radius of the hemisphere is 6 cm and

10 cm
the height of the cone is 10 cm. Find the volume of the
model in terms of r.

Solution Volume of the model Diagram Clue


= volume of the hemisphere + volume of the cone Required volume:
= b 21 # 43 # r # 63 + 1 # r # 62 # 10l cm3
3 6 cm
3
= (144r + 120r) cm

10 cm
6 cm +
= 264r cm3

Instant Drill 9
The solid in the figure consists of a hemisphere
and a cylinder with a common base. The base 3m
radius and the height of the cylinder are both 3m

3 m. Find the volume of the solid in terms of r.

➥ Ex 10C 11, 12

Example 10 In the figure, there is some water in a right cylindrical 12 cm

Level 2 container with a base diameter of 12 cm. If an iron sphere


Cross-topic is immersed in water, the water level rises by 1 cm. Find
the radius of the iron sphere. 1 cm

Solution Let r cm be the radius of the iron sphere.


2
Volume of the water that has risen = r # a 12
2
k # 1 cm3 Diagram Clue
3
= 36r cm
12 cm
Volume of the iron sphere = volume of the water that has risen
4 rr3 = 36r
3
1 cm
r3 = 27
r=3
` The radius of the iron sphere is 3 cm. The water that has risen can
be treated as a cylinder
with base diameter 12 cm
and height 1 cm.
Mensuration (III) 10.31

Instant Drill 10
A copper sphere of radius 5 cm is immersed in water inside a right
cylindrical container. If the base radius of the container is 7 cm, find
the rise in water level, correct to 1 decimal place.
➥ Ex 10C 17, 19

B Surface Area of a Sphere


If the radius of a sphere is r, then

surface area of a sphere = 4rr2 ◀ Please refer to the IT


Exploration ‘Verifying the
r Surface Area Formula from the
Volume Formula for Sphere’ in
Appendix III on P.A22.

S T E M Zone
Quick
Example Surface area of the volleyball in the figure
2
= 4 # r # a 21 k cm2
2
2
= 441r cm
21 cm

Quick Quiz A soap bubble minimizes


its surface area due to
Find the surface area of the sphere in the figure in terms of r.
surface tension. Since a
sphere has the smallest
surface area for a given
8m volume, soap bubbles are
usually spherical in shape.
#minimal surface
#surface tension

Example 11 The picture shows half of a watermelon. Suppose it is a


Level 1 hemisphere with a total surface area of 432r cm2. Find
the radius of the watermelon.

Solution Let r cm be the radius of the watermelon.


Total surface area
= area of circular plane + curved surface area
= brr2 + 21 # 4rr2l cm2
= 3rr2 cm2
` 3rr2 = 432r
r2 = 144
r = 12
` The radius of the watermelon is 12 cm.
10.32 Chapter 10

Instant Drill 11
The surface area of a tennis ball is 137 cm2. Find its diameter, correct
to 2 significant figures.
➥ Ex 10C 4, 6, 7

Example 12 A capsule is in the shape of a right cylinder with two


Level 2 hemispheres at both ends as shown. The diameter of 4 mm

Cross-topic each hemisphere is 4 mm. The volume of the space in the


capsule is 140 3
3 r mm . Find
(a) the length of the capsule,

(b) the total surface area of the capsule.


(Give the answer correct to 3 significant figures.)

Solution (a) Let h mm be the height of the cylinder. Diagram Clue


Base radius of the cylinder = radius of the hemisphere 2 mm 2 mm

= 24 mm
= 2 mm
h mm
Volume of the space in the capsule
= 2 # volume of the space in the hemisphere +
volume of the space in the cylinder
= volume of the space in the sphere + ◀ Two identical hemispheres
volume of the space in the cylinder form a sphere.

= b 43 # r # 23 + r # 22 # hl mm3

= b 32
3 r + 4rhl mm
3

` 32 140
3 r + 4rh = 3 r

4rh = 108
3 r
h=9
` Length of the capsule = (2 + 9 + 2) mm
= 13 mm

(b) Total surface area of the capsule


= 2 # curved surface area of the hemisphere +
curved surface area of the cylinder
= surface area of the sphere +
curved surface area of the cylinder
= (4 # r # 22 + 2 # r # 2 # 9) mm2 ◀ Recall:
= 163 mm2 , cor. to 3 sig. fig. Curved surface area of a right
cylinder = 2rrh
Mensuration (III) 10.33

Instant Drill 12
The wooden chip in the figure consists of a
right cylinder and a hemisphere with a common 3 cm
base. The volume of the chip is 36r cm3. Find
(a) the height of the chip,
(b) the total surface area of the chip in terms of r.
➥ Ex 10C 16, 20, 22

Class Practice 10.3


[In this exercise, unless otherwise stated, give the answers correct to 3
significant figures if necessary.]
1. Find the volume and total surface area of each of the following sphere
and hemisphere.
(a) (b)

16 cm
10 cm

2. The surface area of a sphere is 100r mm2. Find


(a) the radius of the sphere,

(b) the volume of the sphere.

3. The solid in the figure consists of a right circular cone and a hemisphere
with a common base.

10 m 8m

6m

Find
(a) the volume of the solid,

(b) the total surface area of the solid.


(Give the answers in terms of r.)
10.34 Chapter 10

lae
Exercise 10C For
mu

[In this exercise, unless otherwise stated, give the answers correct to • Volume of a sphere = 4 rr3
3
3 significant figures if necessary.] • Surface area of a sphere
= 4rr2
Level 1

For each sphere or hemisphere in Nos. 1–3, find the


(a) volume,

(b) total surface area.


(Give the answers in terms of r.)
1. 2. 3.
9 cm
1.2 m
15 cm

Find the unknown in each of the following spheres. [Nos. 4–5]


4. 5.

r mm rm

➥ Example 11

surface area = 64r mm2 volume = 32 r m3


3

Find the unknown in each of the following hemispheres. [Nos. 6–7]


6. 7. dm
d cm

curved surface area = 200r cm2 total surface area = 147r m2

8. The volume of a sphere is 36r m3. Find its diameter.

9. In the figure, the circumference of the base of a hemisphere is 20r cm. circumference
= 20r cm
Find its volume.
Mensuration (III) 10.35

10. The figure shows a large spherical plastic balloon with a metal
coating on its surface. If its diameter is 24.5 m, find the area of the
coating.

11. In the figure, a sphere of radius 9 cm can just fit into a right
cylindrical container of base radius 9 cm and height 18 cm. Find the
9 cm
volume of the remaining space in the container in terms of r. ➥ Example 9 18 cm

12. The figure shows a gas container. The shapes of its upper part and
lower part are a hemisphere and a cylinder respectively. Their base
5m
radii are 5 m and the height of the cylinder is 8 m. Find the capacity
of the container.
8m

13. The solid in the figure consists of a right circular cone and a
hemisphere with a common base. Find

12 cm
(a) the slant height of the cone,

(b) the total surface area of the solid in terms of r.


5 cm

2
14. The surface area of a globe is 144r cm . Find

(a) its radius,

(b) its volume in terms of r.

15. 8 small solid metal spheres each of radius 2 cm are melted and
recast into a large solid metal sphere. Find
(a) the radius of the large sphere,

(b) the surface area of the large sphere in terms of r.


10.36 Chapter 10

Level 2

16. The solid in the figure is formed by two hemispheres. If the radius of 3 cm

the small hemisphere is 3 cm and the radius of the large hemisphere


is 5 cm, find
5 cm
(a) the curved surface area of the small hemisphere,

(b) the curved surface area of the large hemisphere,

(c) the total surface area of the solid. ➥ Example 12


(Give the answers in terms of r.)

17. Refer to the figure. A solid chocolate sphere of radius 4 cm is


melted to form a solid right circular chocolate cone of height 9 cm.
Find the base radius of the circular chocolate cone.

4 cm 9 cm

➥ Example 10

18. The figure shows an ice cream ball in the shape of a sphere with
radius 9 mm. The surface of the ice cream ball is evenly coated with 2 mm
9 mm
chocolate of thickness 2 mm.
(a) Find the volume of the chocolate coating.
3
(b) If each mm of chocolate weighs 0.03 g, find the weight of the
chocolate coating.

19. In the figure, there is some water in a right cylindrical container


of base diameter 20 cm. 3 iron spheres of radius 2 cm each are
immersed in water. What is the rise in water level?
20 cm

10 cm
20. The figure shows a container in the shape of a hemisphere with
2 cm
uniform thickness. Its internal radius is 10 cm and its thickness is
2 cm. Find the total surface area of the container in terms of r.
Mensuration (III) 10.37

21. The figure shows a steel cake mould in the shape of a hemisphere. 10 cm

The mould is 1 mm thick with an external diameter of 10 cm. If each 1 mm

cm3 of steel weighs 8 g, what is the total weight (in kg) of steel
required to make 5 000 cake moulds?

22. The solid in the figure consists of a right circular cone and a
hemisphere with a common base. The slant height of the circular
25 cm
cone is 25 cm and its curved surface area is 56 of the curved surface
area of the hemisphere. Find
(a) the radius of the hemisphere,

(b) the total surface area of the solid in terms of r,

(c) the volume of the solid in terms of r.

23. In the figure, the total surface area of the right cylinder is
the same as the total surface area of the hemisphere.
6 cm 6 cm
(a) Find the height of the cylinder.
Explain (b) Is the volume of the cylinder also the same as the
volume of the hemisphere? Explain your answer.

24. In the figure, a solid metal sphere rests in a rectangular tray. The
radius of the metal sphere is 3 cm. The width and the length of the 9 cm
3 cm
base of the tray are 15 cm and 20 cm respectively. The depth of
water in the tray is 3 cm.
15 cm

(a) Find the volume of water in the tray.


20 cm
(b) If the height of the tray is 9 cm, find the volume of water
(in cm3) needed to be poured into the tray so that the metal
sphere is just submerged.

25. In the figure, a right circular cone of height 15 cm holds some ice
cream. The ice cream above the cone is a hemisphere and the ice
cream inside the cone occupies 1 of the space in the cone. Assume
6 15 cm
that the volume of the ice cream is the same after melting and
occupies 2 of the volume of space in the cone. Find the base radius
3
of the cone.
10.38 Chapter 10

Level 3

26. In the figure, a right circular cone of base radius 4 cm and height
8 cm is crafted from a wooden sphere.
8 cm
(a) Find the radius of the sphere.

(b) What percentage of the volume of the sphere is the volume of


the cone? 4 cm

27. A container in the shape of an inverted right circular cone of base


radius R cm and height 24 cm is placed vertically. Some water
is poured into the container such that the depth of water in the
container is 12 cm. It is given that the base area of the container is
4 times the surface area of a solid metal sphere of radius r cm.
(a) Find r : R.

(b) It is given that the volume of the water inside the container is
144r cm3.
(i) Find the values of r and R.
Explain (ii) Some metal spheres of radius r cm each are put into the

container and they are immersed in water. Is it possible


that the depth of water increases by 5 cm? Explain your
answer.

10.5 Relations among Lengths, Areas and Volumes


of Similar Figures
A More about Similar Plane Figures
In Book 2B Chapter 9, we learnt that two plane figures having the
same shape are called similar figures (as shown on the right), and all
their corresponding lengths are proportional. In this section, we will
investigate the relation between the ratio of the lengths and the ratio
of the areas of similar plane figures.
Mensuration (III) 10.39

Class Activity 10.3


For each question in the table below, figures X and Y are similar, where the lengths marked with the
same colour are the corresponding lengths. Complete the table.

Ratio of corresponding
Similar figures Ratio of areas
lengths
2
1. Area of X
Radius of X
= = r#2
Radius of Y Area of Y ( )
3
2
2 =b l

circle X circle Y

2. 3a Length of X Area of X ( )
= = 1 =
Length of Y ( ) Area of Y ( )( )
2
=b l
Width of X 1
3b = =
a Width of Y ( )

rectangle X rectangle Y

From the results of Class Activity 10.3, we have:


·

For two similar plane figures, the ratio of their areas is equal to the
square of the ratio of any two corresponding lengths. ◀ The corresponding lengths here
refer to the lengths, widths,
heights, radii, perimeters,
If the areas of two similar plane figures are A1 and A2, and the two circumferences, etc. of the
corresponding lengths are l1 and l2, then figures.

A1 = l1 2
c m
A2 l2

Quick Quick Quiz


Example In the figure, 3ABC + 3XYZ.

= a 15
9 cm k
Area of 3ABC 2
X In the figure, ABCD + PQRS.
Area of 3XYZ cm area of ABCD
Find .
= a 53 k
2 area of PQRS
A
mc

P Q
15

A B
m
9c

9
12

= 25
10

cm
cm

B C Y Z
D C S R
10.40 Chapter 10

Example 13 In the figure, the ratio of the lengths of sides of two


Level 1 regular pentagons is 3 : 4. If the area of the large pentagon
is 192 m2, find the area of the small pentagon.

Solution Let x m2 be the area of the small pentagon. Clue


2 2
x m = a3k Regular polygons with the
192 m
2 4
same number of sides are
x = 9 similar figures.
192 16
9
x = 16 # 192
= 108
` The area of the small pentagon is 108 m2.

Instant Drill 13
It is given that the ratio of the radii of two similar sectors is 2 : 3. If
the area of the small sector is 28 m2, find the area of the large sector.
➥ Ex 10D 3–6

Example 14 The figure shows the plan of a club house of an estate


Level 1 with scale 1 : 500. If the area of the plan is 24 cm2, what
is the actual area of the club house in m2?

Solution Let x m2 be the actual area of the club house.


Area of the plan 1 2
= a 500 k
Actual area
2
24 cm = a 1 k2
xm
2 500
2
24 cm = 1 ◀ 1 m = 100 cm
2 250 000
x # 100 # 100 cm 1 m2 = 100 # 100 cm2
x = 24 # 250 000
100 # 100
= 600
` The actual area of the club house is 600 m2.

Instant Drill 14
The actual area of a public place is 7 200 m2 and its area on a plan is ➥ Ex 10D 13

2 cm2. If the scale of the plan is 1 : k, find the value of k. Public Exam 10, 17
Questions

For an application of similar plane figures,


please refer to the Inquiry and Investigation
‘Areas of China and Singapore’ in Appendix IV
on P.A28.
Mensuration (III) 10.41

Example 15 In the figure, ABCD is a parallelogram. F is a point on AD. E

Level 2 BA produced and CF produced intersect at E. If


EF : FC = 2 : 1 and the area of 3DEF is 6 cm2, find
(a) the area of 3CDF, A D
F
(b) the area of 3EAF,
B C
(c) the area of ABCF.

Solution (a) Clue


3DEF and 3CDF are two triangles with ‘same height
but different bases’. The ratio of their areas is equal to
the ratio of the lengths of their bases.

Let x cm2 be the area of 3CDF.


Area of 3DEF D
= EF same
height
Area of 3CDF FC
2 E C
6 cm = 2 F
2 1
x cm different bases
x=3
` The area of 3CDF is 3 cm2.
2
(b) Let y cm be the area of 3EAF.
a 3EAF + 3CDF (AAA) E

= a CF
EF 2
k
Area of 3EAF l 2
` ◀ A1
= c 1m
Area of 3CDF A2 l2
2
= a 21 k
y cm 2
2 D
3 cm A
F
y
= 4
3 1
C
y = 12
` The area of 3EAF is 12 cm2.
2
(c) Let z cm be the area of ABCF.
a 3EAF + 3EBC (AAA) E

= a EC k
EF 2
Area of 3EAF
`
Area of 3EBC
a 2 k2
2
12 cm = ◀ EC = EF + FC
2 2 2+1 A F
12 cm + z cm = 2FC + FC
12 = 4 = (2 + 1) FC
12 + z 9 B C

48 + 4z = 108
4z = 60
z = 15
` The area of ABCF is 15 cm2.
10.42 Chapter 10

Instant Drill 15
In the figure, PQRS is a rectangle. T and U P Q

are points on PS and QR respectively such


T U
that TU # SR. TU and QS intersect at V, V

QV : VS = 1 : 3 and the area of URSV is


45 cm2. Find
(a) the area of 3QUV,
S R
(b) the area of 3STV, ➥ Ex 10D 18, 19, 24

(c) the area of PQVT. Public Exam 9, 12, 13, 15, 16, 19
Questions

B Similar Solids
In daily life, have you seen the model of an aeroplane?

A380 A380 AIRBUS

an aeroplane and its model

In the figure, the aeroplane and its model are of the same shape.
In Mathematics, two solids of the same shape are called similar solids.
All corresponding lengths of two similar solids are proportional and all
their pairs of corresponding angles are equal.
In this section, we are going to investigate the relation between the ratio
of lengths and the ratio of areas/ratio of volumes in similar solids.

Class Activity 10.4


For each question in the table below, solids M and N are similar, where the lengths marked with the
same colour are the corresponding lengths. Complete the table.

Ratio of Ratio of
Similar solids Ratio of areas
corresponding lengths volumes

1. Length of each side of M Total surface area of M Volume of M


Length of each side of N Total surface area of N Volume of N
3
2
( ) 6#( ) ( )
1 = = 2
=
( ) 6#( ) ( )
3
1 2 3
1 3 =b l =b l
cube M cube N

similar solids 相似立體


Mensuration (III) 10.43

Ratio of Ratio of
Similar solids Ratio of areas
corresponding lengths volumes

2. Radius of M Surface area of M Volume of M


Radius of N Surface area of N Volume of N

3 = 4r # ( )
2
( )
2
= =
( ) ( )
2 3
sphere M sphere N =b l =b l

From the results of Class Activity 10.4, we have:

For two similar solids,


(a) the ratio of the areas of any two corresponding surfaces is equal ◀ The areas of corresponding
to the square of the ratio of any two corresponding lengths, surfaces here refer to the base
areas, curved surface areas,
(b) the ratio of the volumes is equal to the cube of the ratio of any total surface areas, etc. of the
two corresponding lengths. solids.

If the volumes of two similar solids are V1 and V2, the areas of the two
corresponding surfaces are A1 and A2, and the two corresponding lengths
are l1 and l2, then

l 2
c 1m
A1
1. A2
=
l2
l 3
c 1m
2. V1
V2 =
l2

Quick Quick Quiz


Example The figure shows two similar pyramids A and B, where the given
heights are two corresponding lengths. The figure shows two similar
circular cones P and Q, where
the given lengths are two
height = 4 cm height = 5 cm
corresponding lengths.

A Q
B P

= a 45 k = 16
Total surface area of A 2 3 cm 4 cm
Total surface area of B 25
Find

= a 45 k = 125
Volume of A 3 curved surface area of P
64 (a) ,
Volume of B curved surface area of Q
volume of P
(b)
volume of Q
.
10.44 Chapter 10

S T E M Zone
Assume the bodies of the adult polar bear and
its cub in the photo are similar. Then we have:
h1
Adult polar h2
Cub
bear

Surface area 4A A
h1 : h2 = 2 : 1
Volume 8V V
Surface area A A
Volume 2V V

surface area
The adult polar bear has a lower value of ,
volume
and it loses heat more slowly than its cub in cold climates.
#heat loss

Example 16 In the figure, the heights of two similar prisms 4 cm


Level 1 are 4 cm and 6 cm. If the volume of the small
prism is 24 cm3, find the volume of the large 6 cm

prism.

Solution Let x cm3 be the volume of the large prism. Clue


3 3
x cm = a 6 k The heights of the two
24 cm
3 4
similar prisms are two
x = 27 corresponding lengths.
24 8

x = 27
8 # 24
= 81
` The volume of the large prism is 81 cm3.

Instant Drill 16
The ratio of the heights of two similar cylinders is 1 . If the curved
2
2
surface area of the large cylinder is 50 cm , find the curved surface
area of the small cylinder.
➥ Ex 10D 9–12
Mensuration (III) 10.45

Example 17 In the figure, the ratio of the volumes of two


Level 2 hemispheres P and Q is 1 : 8.
(a) Find the ratio of the circumference of the base
of P to the circumference of the base of Q. P Q
2
(b) If the total surface area of Q is 128 mm , find
the total surface area of P.

Solution Let a and b be the circumferences of the bases of P and Q


respectively.
3 volume of P
(a) a a k = Clue
b volume of Q
3 All spheres (hemispheres)
aak = 1
b 8 are similar solids.
a = 1
b 2
` The required ratio is 1 : 2.
2
(b) Let x mm be the total surface area of P.
x mm = a a k2
2
2 b
128 mm
x = a 1 k2
128 2

x = 1 # 128
4
= 32
` The total surface area of P is 32 mm2.

Instant Drill 17
The ratio of the surface areas of two spheres A and B is 25 : 36.
(a) Find the ratio of the radii of A and B.
3
(b) If the volume of A is 50 m , find the volume of B.
➥ Ex 10D 20, 21

Class Practice 10.4


1. The figure shows two regular octagons A and B. Find the ratio of the
areas of A and B.
B

2 cm 7 cm
10.46 Chapter 10

2. The figure shows two similar trapeziums ABCD H G


perimeter
and EFGH. If the area of ABCD is 8 cm2, find perimeter
D C
= 18 cm
= 12 cm
the area of EFGH.
A B E F

3. It is known that the ratio of the areas of two circles is 16 : 25. If the
radius of the large circle is 15 mm, find the radius of the small
circle.

4. The figure shows two hemispheres X and Y. Find the ratios of 3 cm 4 cm

(a) the curved surface areas of X and Y,

(b) the volumes of X and Y. X


Y

5. In the figure, the ratio of the heights of two similar pyramids P


and Q is 2 : 5.
2
(a) If the total area of all lateral faces of P is 32 cm , find
the total area of all lateral faces of Q.
3
(b) If the volume of Q is 500 cm , find the volume of P. P Q

Exercise 10D For


m ulae

Level 1
• For two similar plane figures,
l 2
In each of the following, figure A and figure B are similar, where the = c 1m
A1
A2 l2
marked lengths are corresponding lengths. Find the ratio of the area
of A to that of B. [Nos. 1–2] • For two similar solids,
l 2
= c 1m
A1
1.
1. 8m A2 l2
4m
l 3
= c 1m
V1
2.
V2 l2
A
B

2. 7 cm

3 cm

A B
Mensuration (III) 10.47

Find the unknown in each of the following pairs of similar plane figures, where the marked lengths are
corresponding lengths. [Nos. 3–6]
3. 4. 8m
m
6m
m
15 area = 72 mm2
1102
ccm
cm area = A mm2
m
➥ Example 13
area = 32 cm2 area = A cm2

5. 6.
xm
10 m
x cm

8 cm

area = 175 cm2 area = 28 cm2 area = 75 m2 area = 108 m2

In Nos. 7–8, solid A and solid B are similar, where the marked lengths are corresponding lengths. Find
the ratio of
(a) the curved surface area of A to the curved surface area of B,

(b) the volume of A to the volume of B.

7. 8.

15 cm 18 cm
9 cm
4 cm

A B A B

Find the unknown in each of the following pairs of similar solids, where the marked lengths are
corresponding lengths. [Nos. 9–12]
9. 10.
curved surface
curved surface
area = 63 cm2
area = c cm2 15 cm
12 cm

3 cm 2 cm

➥ Example 16
volume = v cm3 volume = 1 500 cm3

11. 11 cm 12.
y cm
x cm
2 cm

total surface area total surface area volume = 24 cm3 volume = 81 cm3
= 539 cm2 = 1 331 cm2
10.48 Chapter 10

13. Lantau South Country Park is the largest country park in Hong
Kong. Its actual area is 56 400 000 m2. Find the area (in cm2) of
the country park on a map of scale 1 : 50 000. ➥ Example 14

14. In the figure, D and E are points on AB and AC respectively such A

that DE # BC. If AD : DB = 3 : 1 and the area of 3ADE is 18 cm2,


find
(a) AD : AB, D E

(b) the area of 3ABC, B C

(c) the area of BCED.

15. Pyramid H is similar to pyramid K. If the volume of H is 27 times


that of K, find the ratio of
(a) the height of H to that of K,

(b) the total surface area of H to that of K.

2 2
16. The base areas of two similar circular cones are 25r cm and 36r cm .
Find the ratio of
(a) the curved surface area of the large cone to that of the small
cone,
(b) the height of the large cone to that of the small cone,

(c) the volume of the large cone to that of the small cone.

17. After melting a gold prism of height 10 cm, how many gold prisms
can be recast if each prism is of height 5 cm and is similar to the
original one?

Level 2 &
18. In the figure, ABCD and EFGD are two rectangles and A E
D
ABCD + EFGD. If DG : GC = 1 : 2, find
G
F
(a) DG : DC,

(b) the area of rectangle EFGD:the area of hexagon ABCGFE. B C


➥ Example 15
Mensuration (III) 10.49

19. In the figure, D, E and F are points on AB, AC and BC respectively A

such that DE # BC and BA # FE. If BF : FC = 1 : 2 and the area


of 3ADE is 2 cm2, find the area of D E

(a) 3EFC,

(b) parallelogram BFED.


B F C

20. The snowman in the figure consists of two spheres. The ratio of the
volumes of the two spheres is 8 : 27.
(a) Find the ratio of the diameter of the small sphere to the
diameter of the large sphere.
2
(b) If the surface area of the large sphere is 1.8r m , find the
surface area of the small sphere in terms of r. ➥ Example 17

21. In the figure, XABCD and YPQRS are two similar pyramids. Y

It is given that ABCD a PQRS and BC : QR = 3 : 4.


X
2
(a) If the area of quadrilateral ABCD is 45 cm , find the
D S
area of quadrilateral PQRS. C
R
3 A
(b) If the volume of YPQRS is 320 cm , find the volume of B
P
XABCD. Q

2
22. The surface area of a volleyball is 1 400 cm and its volume is
5 000 cm3. If the diameter of a tennis ball is 30% of that of the
volleyball, find
(a) the surface area of the tennis ball,

(b) the volume of the tennis ball.

23. If the surface area of a sphere increases by 69%, find the


percentage increase in
(a) the radius of the sphere,

(b) the volume of the sphere.

24. In the figure, ABCD is a rectangle. E and F are points on AD and A E D


BC respectively such that BA # FE. BD and EF intersect at G. If
EG : GF = 3 : 1, find the area of 3BFG:the area of ABGE.
G
B F C
10.50 Chapter 10

25. Lilian melts a solid right circular chocolate cone of base radius R mm and height 60 mm to form
6 identical solid right circular chocolate cones of base radii r mm as shown below.

60 mm

R mm

r mm

It is given that the base area of the large chocolate is 4 times that of each small chocolate.
(a) Find R : r.

(b) Find the height of each small chocolate.


Explain (c) Lilian claims that the large chocolate is similar to each small chocolate. Do you agree? Explain
your answer.
height of the large chocolate
(Hint: If ! Rr , then the large chocolate is not similar to each small
height of each small chocolate
chocolate.)

Level 3

26. In the figure, PQRS is a parallelogram. U, V and W are points Q W R

on PS, PQ and QR respectively such that PU : US = 3 : 2,


QV : VP = 2 : 1 and QW : WR = 1 : 3. PR and WU intersect at
V X
X. If the area of 3QWV is 20 cm2, find the area of 3PXU.
P U S

27. In Fig. A, ABCDEF is a wooden block in the shape of a right prism. Its base is a right-angled
triangle ABC. It is given that BC = 15 cm, AB = 12 cm and CD = 10 cm.
E E

S R
A A
F D F D
P Q

B C B C

Fig. A Fig. B

(a) Find the volume of the block ABCDEF.

(b) As shown in Fig. B, the block ABCDEF is cut by the plane PQRS which is parallel to the plane
BCDF into two blocks APQRES and BCQPSFDR. It is given that AP = 4 cm.
(i) Find the volume of the block APQRES.
Explain (ii) Are the blocks APQRES and ABCDEF similar? Explain your answer.
(Hint: Compare the ratio of corresponding lengths.)
Mensuration (III) 10.51

10.6 Frustums of Pyramids and Circular Cones


When a pyramid is cut by a plane parallel to its base, the removed part at
the top is a small pyramid and the remaining part is called a frustum. The
two parallel planes on the frustum are called the upper base and the lower
base. The perpendicular distance between these two bases is the height of
the frustum (see Fig. 7).

small pyramid

a plane parallel upper base and


to the base

height frustum of the are similar solids.


pyramid

lower base

Fig. 7

Similarly, we can also get a frustum of a circular cone (see Fig. 8).

small circular cone

a plane parallel
to the base
and
upper base

are similar solids.


height
frustum of the
circular cone
right circular cone

lower base

Fig. 8

In this section, we will discuss the frustums of pyramids and the frustums Interesting Maths
of circular cones. The following are examples
of frustums.

El Castillo Sand bucket

frustum 平截頭體
10.52 Chapter 10

A Volumes of Frustums of a Pyramid and a Circular Cone


We learnt that a frustum of a pyramid is the remaining part when a
smaller pyramid is removed from a larger pyramid. Thus, we have:

Volume of frustum C
= volume of pyramid A - volume of pyramid B = -

C A B

i
Example 18 Find the volume of the frustum of the pyramid in the
a
3m
Level 1 figure. 4m
6m

12 m
The upper base and the
Solution Volume of the larger pyramid lower base are squares.

= 1 # 122 # (3 + 6) m3 ◀ Volume of a pyramid


3
= 1 # base area # height
= 432 m3 3

Volume of the smaller pyramid


= 1 # 42 # 3 m3
3
= 16 m3
Volume of the frustum
= volume of the larger pyramid - volume of the smaller pyramid
= (432 - 16) m3
= 416 m3

Instant Drill 18
Find the volume of the frustum of the
pyramid in the figure. 9 cm

5 cm
9 cm

10 cm
The upper base and the
lower base are squares.
➥ Ex 10E 1, 2
Mensuration (III) 10.53

Similarly, for a frustum of a circular cone:

Volume of frustum C = -
= volume of circular cone A -
volume of circular cone B
C A B

Example 19 The picture shows a pudding in the A


Level 1 shape of the frustum of a right circular 2 cm
cone. The dimensions of the pudding
are shown in the figure. Find B M C

4.5 cm
(a) the length of AM,
D N E
(b) the volume of the pudding in terms of r. 4 cm

Solution (a) Let AM = x cm, A

then AN = (x + 4.5) cm.


x cm
a 3AMC + 3ANE (AAA)
AM = MC M C
` 2 cm
AN NE
4.5 cm
x = 2
x + 4.5 4
N E
4x = 2x + 9 4 cm

2x = 9
x = 4.5
` AM = 4.5 cm

(b) Volume of circular cone ADE

= 1 # r # 42 # (4.5 + 4.5) cm3


3
= 48r cm3
Volume of circular cone ABC
= 1 # r # 22 # 4.5 cm3
3
= 6r cm3
Volume of the pudding
= volume of circular cone ADE - volume of circular cone ABC
= (48r - 6r) cm3
= 42r cm3
10.54 Chapter 10

Instant Drill 19
The figure shows the frustum of a right A 2 cm
circular cone. The radii of the upper B P C
base and lower base are 2 cm and 8 cm
9 cm
respectively. Its height is 9 cm. Find
(a) the length of AP, D Q
8 cm
E ➥ Ex 10E 9

(b) the volume of the frustum in terms of r. Public Exam 14


Question

Example 20 The figure shows the frustum ABCDEFGH of an 20 cm


B 24 cm A

Level 2 inverted right pyramid. Its upper base and lower C D

base are rectangles. The height of pyramid VABCD 15 cm G F


H E 16 cm
is 16 cm. Find
18 cm
(a) the height of the frustum,
V
(b) the volume of the frustum.

Solution Refer to the notations in the figure. Clue


B A In the figure, draw 3VXM
N Y
C D and 3VYN, where
G
M F (i) M and N are the
X
H E mid-points of GH and
BC respectively.
V (ii) VY and VX are the
heights of the large
(a) Let h cm be the height of the frustum. and small pyramids
VX = (16 - h) cm respectively.

XM = 1 # FG = 1 # 18 cm = 9 cm
2 2

YN = 1 # AB = 1 # 24 cm = 12 cm
2 2
a 3VXM + 3VYN (AAA)
` VX = XM 12 cm
N Y Do’s and Don’ts
VY YN
h cm
16 - h = 9 9 cm Do not make the following
M X
16 12 16 cm mistakes when setting up
192 - 12h = 144 equations.
48 = 12h (i) h = 9

h=4
V 16 12 ✗
(ii) 16 - h = 9
` The height of the frustum is 4 cm. h 12 ✗
Mensuration (III) 10.55

(b) Volume of pyramid VABCD

= 1 # (24 # 20) # 16 cm3


3
= 2 560 cm3
Volume of pyramid VEFGH
= 1 # (18 # 15) # (16 - 4) cm3
3
= 1 080 cm3
Volume of the frustum
= volume of pyramid VABCD - volume of pyramid VEFGH
= (2 560 - 1 080) cm3
= 1 480 cm3

Alternative Method

For (b):
1. Find the volume of the pyramid VABCD. Ans

2. (a) Find HE . Ans (a)


CD
(b) Hence, using the properties of similar solids, find
volume of VEFGH
. (b)
volume of VABCD
3. Using the results above, find the volume of the pyramid VEFGH. Ans

4. Hence, find the volume of the frustum. Ans

Instant Drill 20
The figure shows a frustum of a right V

pyramid. Its upper base and lower base


are squares of sides 8 cm and 12 cm E H
respectively. Its height is 3 cm. Find D
F C 3 cm
8 cm G
(a) the height of the pyramid VEFGH,
(b) the volume of the frustum. A 12 cm B
➥ Ex 10E 12, 13
10.56 Chapter 10

Example 21 In the figure, a right pyramid C is divided into 3 cm


Level 2 a small pyramid X similar to C and a frustum
X
Explain Y. The heights of X and Y are 3 cm and 5 cm
respectively. 5 cm

= a 3 k = 27 .
volume of X 3
(a) Jackson claims that Y
volume of Y 5 125 C

Do you agree? Explain your answer.


3
(b) If the volume of Y is 970 cm , find the volume of X.

Solution Let VX and VY be the volumes of X and Y respectively.


(a) Height of C = (3 + 5) cm = 8 cm
Volume of C = VX + VY
a X and C are similar.
3
=c m
Volume of X height of X l 3
= c 1m
` ◀ V1
Volume of C height of C V2 l2
a 3 cm k
VX 3
V X + VY = 8 cm
VX 27
V X + VY = 512
512VX = 27VX + 27VY
485VX = 27VY
VX 27
VY = 485
Volume of X 27
i.e. = 485 ! 27
Volume of Y 125
` The claim is disagreed.
V 27
(b) From (a), VX = 485
Y

VX 27
3
= 485
970 cm
27
VX = 485 # 970 cm3
= 54 cm3
` The volume of X is 54 cm3.

Instant Drill 21
In the figure, an inverted right circular
cone X is divided into a frustum P and P
a small circular cone Q similar to X,
Q
where the ratio of the heights of P and X
Q is 3 : 2. Find the ratio of the volumes
of P and Q.
➥ Ex 10E 14
Mensuration (III) 10.57

B Total Surface Areas of Frustums of a Pyramid and a Circular Cone


Consider the total area of all lateral faces of a frustum of a pyramid. We have:

Total area of all lateral faces of frustum C


= total area of all lateral faces of pyramid A - total area of all lateral faces of pyramid B

= -
C A B

If the areas of the upper base and lower base of the frustum are added to the above result, we have:
upper base
Total surface area of a frustum of a pyramid lateral
face
area of area of total area of
= + +
the upper base the lower base all lateral faces

Note: Each lateral face of a frustum of a pyramid is a trapezium. lower base

When finding the total area of all lateral faces of a frustum of a


pyramid, we may consider the total area of these trapeziums.

Example 22 Find the total surface area of the frustum of the right 7 cm
Level 1 pyramid in the figure. 6 cm

7 cm
Solution Total area of all lateral faces of the larger pyramid
= 4 # 9 21 # 12 # (7 + 7)C cm2 ◀ 4 # area of large triangle 12 cm

= 336 cm2 The upper base and the


lower base are squares.
Total area of all lateral faces of the smaller pyramid
= 4 # a 1 # 6 # 7k cm2 ◀ 4 # area of small triangle
2
= 84 cm2
10.58 Chapter 10

Total area of all lateral faces of the frustum


= total area of all lateral faces of the larger pyramid -
total area of all lateral faces of the smaller pyramid
= (336 - 84) cm2
= 252 cm2
Total surface area of the frustum
= area of the small square + area of the large square +
total area of all lateral faces of the frustum
= (62 + 122 + 252) cm2
= 432 cm2

Alternative Method
Each lateral face of the frustum is a trapezium.
1. Find the area of each lateral face of the frustum. Ans

2. Find the total area of all lateral faces of the frustum. Ans

3. Find the total surface area of the frustum. Ans

Instant Drill 22
The figure shows a frustum of a right 13 cm
15 cm
pyramid. Its upper base and lower base
10 cm
are rectangles of 18 cm by 10 cm and
18 cm 15 cm
36 cm by 20 cm respectively. Find the
13 cm
total surface area of the frustum. 20 cm
36 cm

The upper base and the


lower base are rectangles.
➥ Ex 10E 5, 8(a)

Similarly, for a frustum of a circular cone:

Curved surface area of frustum C


= curved surface area of circular cone A - curved surface area of circular cone B

= -

C A B
Mensuration (III) 10.59

If the areas of the upper base and lower base of the frustum are added to
the above result, we have:
upper base

Total surface area of a frustum of a circular cone curved


surface
area of area of curved surface
= + +
the upper base the lower base area
lower base

Example 23 The figure shows a frustum of a right circular cone. V

Level 1
(a) Find the lengths of VQ and VS. 12 cm
5 cm
P N Q
(b) The curved surface of the frustum is now painted.
If the painting cost is $0.2 per cm2, find the total
cost. 15 cm
(Give the answer correct to 3 significant figures.)
R M S

Solution (a) In 3VNQ, V

VQ = VN2 + NQ2 (Pyth. theorem) 12 cm


2 2
= 12 + 5 cm N
5 cm
Q

= 13 cm
Let VS = x cm. V
a 3VNQ + 3VMS (AAA)
13 cm

` VS = MS
N Q
VQ NQ x cm
5 cm
x = 15
13 5

x = 15 # 13
5
M S
15 cm
= 39
` VS = 39 cm

(b) Curved surface area of the frustum


= curved surface area of circular cone VRS -
curved surface area of circular cone VPQ
= (r # 15 # 39 - r # 5 # 13) cm2
= 520r cm2
Total cost
= $0.2 # 520r
= $327, cor. to 3 sig. fig.
10.60 Chapter 10

Instant Drill 23
The figure shows a frustum of an inverted 8 cm
C K D
right circular cone.
4 cm
(a) Find the lengths of VH and VD. A H B

(b) The surface of the frustum is now painted. 5 cm


If the painting cost is $0.1 per cm2, find the V
total cost.
(Give the answer correct to 3 significant figures.)
➥ Ex 10E 10

Example 24 A rectangular tank of height 12 cm contains some


Level 2 water. A regular pyramid with a square base of side
24 cm
20 cm and height 24 cm is then put into the tank.
Water reaches the top of the tank and does not
12 cm
overflow as shown in the figure.
20 cm
(a) Find the total area of all lateral faces of the
pyramid.
(b) Find the total area of the wet surfaces of the
pyramid.

Solution Refer to the notations in the figure below. Clue


V
In the figure, draw3VXM,
where M is the mid-point of
BC and VX is the height of
the pyramid.
D C

X M
A
B

(a) XM = 1 # AB = 1 # 20 cm = 10 cm V
2 2
In 3VXM,
24 cm
VM = VX2 + XM2 (Pyth. theorem)

= 242 + 102 cm X M
10 cm
= 26 cm
` Area of 3VBC = 1 # BC # VM V
2
1
= # 20 # 26 cm2
2
= 260 cm2
Total area of all lateral faces of the pyramid
B M C
= 4 # area of 3VBC
20 cm
= 4 # 260 cm2
= 1 040 cm2
Mensuration (III) 10.61

2
(b) Let W cm be the total area of the wet parts on the
lateral faces of the pyramid, then the total area of
the dry surfaces of the pyramid is (1 040 - W) cm2.
2
= a 24 - 12 k
(1 040 - W) cm 2
l 2
= c 1m
◀ A1
1 040 cm
2 24 A2 l2
1 040 - W = 1
1 040 4

1 040 - W = 1 # 1 040
4
W = 780
Total area of the wet surfaces of the pyramid
= area of square ABCD + total area of the wet parts
on the lateral faces
= (202 + 780) cm2
= 1180 cm2
Instant Drill 24
A rectangular tank contains some water.
A right circular cone of base radius
16 cm and height 30 cm is then put into
the tank. The depth of water becomes 6 cm.
Find the total area of the wet surfaces of ➥ Ex 10E 15
the circular cone in terms of r. Public Exam
Question 4

Class Practice 10.5


[In this exercise, give the answers correct to 3 significant figures if
necessary.]
1. In the figure, the upper base and the lower base of the frustum of a
pyramid are rectangles. Find its volume. 8 cm
12 cm
cm
6

9 cm
15 cm 10 cm

2. Find the volume of the frustum of the right circular cone in the figure.
10 cm
6 cm
15 cm

9 cm
10.62 Chapter 10

3. The figure shows a frustum of an inverted right circular cone. Find its 12 cm

(a) curved surface area,


8.5 cm
(b) total surface area.

8 cm

Exercise 10E
[In this exercise, unless otherwise stated, give the answers correct to
3 significant figures if necessary.]
Level 1

Find the volume of the frustum of a pyramid or a right circular cone in


each of the following figures. [Nos. 1–4]
1. 2. 15 cm

15 m
8 cm
6m

20 cm
9 cm
5m
9m
8m
12 m

The upper base and the The upper base and the
lower base are rectangles. ➥ Example 18 lower base are squares.

3. 3.2 cm 4.

3 cm
8 cm 2.4 cm 10 cm

4 cm

5 cm
24 cm

5. In the figure, the upper base and the lower base of the frustum of a 25 cm

right pyramid are squares. Find its 30 cm

(a) total area of all lateral faces, 25 cm

(b) total surface area. ➥ Example 22


60 cm
Mensuration (III) 10.63

6. The figure shows a frustum of a right circular cone. Find its


(a) curved surface area, 17 cm 8 cm
34 cm
(b) total surface area.
(Give the answers in terms of r.) 16 cm

7. The figure shows a wooden decoration in the shape of a right circular


A
cone. The curved surface of the upper part (a small right circular 20 cm
cone) is painted in blue, while the curved surface of the lower part 16 cm

(a frustum) is painted in green. Find B M C

(a) the curved surface area of the small right circular cone, D N E
24 cm
(b) the length of AE,

(c) the curved surface area of the whole decoration,

(d) the area of the curved surface in green.


(Give the answers in terms of r if necessary.)

8. In the figure, the upper base and the lower base of the frustum of a
39 cm
right pyramid are squares. Find 36 cm
13 cm
(a) the total surface area of the frustum,

(b) the volume of the frustum. 12 cm

40 cm 30 cm

9. Katie makes a sponge cake. Its shape is a frustum of a right circular A


3 cm
cone in the figure. The height of the cake is 10 cm, and the radii of
B X C
the upper base and the lower base are 3 cm and 9 cm respectively.
Find 10 cm

(a) the length of AX, D Y 9 cm


E

(b) the volume of the sponge cake in terms of r. ➥ Example 19

10. The figure shows a cork. Its shape is a frustum of a right circular V

cone.
(a) Find the length of VY. Y 6 cm Z
M
5 cm
(b) The surface of the cork is now covered with a coating. If the
W N X
cost of coating is $1.8 per cm2, find the total cost. ➥ Example 23 9 cm
10.64 Chapter 10

11. The figure shows a frustum of a right circular cone. Its volume is V

38r cm3. Find the volume of the circular cone VCD in terms of r.
C 4 cm D

A B
6 cm

Level 2

12. The figure shows the frustum ABCDEFGH of a right pyramid. Its V

upper base and lower base are rectangles. The height of pyramid 10 cm
VEFGH is 10 cm. Find 6 cm H
E
F
10 cm G
(a) the height of the frustum,
D C
(b) the volume of the frustum. ➥ Example 20 12 cm
A B
20 cm

13. The figure shows a vessel in the shape of an inverted right pyramid D
18 cm
C
whose base is a horizontal square ABCD. The vessel contains water
of 320 cm3 and the depth of water is h cm. Find B
A
(a) the value of h, S R
8 cm Q
P
(b) the additional volume of water required to fill up the vessel. h cm

14. In the figure, VABCD is a right rectangular pyramid. It is cut by the V

plane EFGH parallel to its base into a small pyramid VEFGH and a F
E
frustum ABCDEFGH. If the ratio of the areas of rectangles ABCD G
A H
and FGHE is 4 : 1, and the volume of the small pyramid VEFGH is D
5 cm3, find the volume of the frustum ABCDEFGH. ➥ Example 21 B
C

15. The figure shows a vessel in the shape of an inverted right circular 12 cm
B X C
cone. The vessel contains some water. Find
(a) the height XA of the vessel, 4 cm

(b) the additional volume of water required to fill up the vessel, D Y E

(c) the area of the dry surface inside the vessel. ➥ Example 24 16 cm

A
Mensuration (III) 10.65

16. In the figure, a right circular cone C is cut by a plane parallel to its
base into a small circular cone X and a frustum Y.

X 3 cm

Y 2 cm
C

Explain (a) Jason claims that:


3 3
(i) Volume of X : volume of Y = 3 : 2
2 2
(ii) Curved surface area of X : curved surface area of Y = 3 : 2
Do you agree? Explain your answer.
2
(b) If the curved surface area of X is 12 cm , find the curved
surface area of Y.

Level 3

17. The figure shows a container in the shape of an inverted right 18 cm

circular cone of base radius 18 cm and height 54 cm. The container


is held vertically and water is poured into the container until the
depth of water is 18 cm. Then, oil is poured into the container above
the water surface until the depth of oil is 9 cm.
54 cm
(a) Find the curved surface area covered with oil. oil 9 cm

3 water
Explain (b) Oil of volume 271r cm is now poured into the container. Daisy 18 cm
claims that the new depth of oil is greater than 12 cm. Do you
agree? Explain your answer.

18. A cake is in the shape of a right circular cone with height 6 cm. The
cake is then cut by a plane parallel to its base and a small circular
cone A at the top is removed. The remaining cake forms a frustum B
of height 4 cm. It is given that the volume of A is 10 cm3.
(a) Find the original volume of the cake.

(b) B is cut by a plane parallel to its base and the upper part of
the frustum is removed. The remaining cake forms a smaller
frustum C. If the volume of C is 190 cm3, find the height of C.
10.66 Chapter 10

Studying
Chapter Summary Tips

Note Example

1. Pyramids
(a) Volume of a pyramid 4 cm 5 cm

= 1 # base area # height


3
(b) Total surface area of a right pyramid 6 cm
= total area of all lateral faces + 6 cm
area of the base right pyramid with a square base
lateral
face
height Volume = 1 # 62 # 4 cm3
3
= 48 cm3

base Total surface area = :4 # a 21 # 6 # 5k + 62D cm2


= 96 cm2

2. Circular Cones

(a) Volume of a circular cone = 1 rr h


2
3 5 cm
4 cm
(b) Curved surface area of a right circular
cone = rrl 3 cm

(c) Total surface area of a right circular


right circular cone
cone = rrl + rr2
where r is the base radius, h is the height and Volume = 1 # r # 32 # 4 cm3
3
l is the slant height. = 12r cm3
Curved surface area = r # 3 # 5 cm2
m
h = 15r cm2
Total surface area = (15r + r # 32) cm2
r = 24r cm2

By Pythagoras’ theorem, l 2 = r2 + h2.

3. Spheres
For a sphere: Volume = 4 # r # 33 cm3
3
(a) volume = 4 rr ,
3 3 cm
3
r
= 36r cm3
(b) surface area = 4rr ,
2 Surface area = 4 # r # 32 cm2
= 36r cm2
where r is the radius.
Mensuration (III) 10.67

Note Example

4. Relations among Lengths, Areas and


Volumes of Similar Figures
X
(a) For two similar plane figures: (a)
If their areas are A1 and A2, and the two A 2 cm
Y
corresponding lengths are l1 and l2, then 1 cm
B
l 2
c 1m .
A1
= C Z
A2 l2
If 3ABC + 3XYZ, then
= a 21 k = 41
2
area of 3ABC
area of 3XYZ
X
(b) For two similar solids: (b)
If their volumes are V1 and V2, the areas P

of the two corresponding surfaces are 2 cm


A 1 and A 2, and the two corresponding 3 cm Y
Q
lengths are l1 and l2, then circular cone A circular cone B
2 3
= c 1 m and V1 = c 1 m .
A1 l V l
If circular cones A and B are similar, then
A2 l2 2 l2
PQ 2
XY = 3

= a 23 k = 49
Curved surface area of A 2

Curved surface area of B


= a 23 k = 27
Volume of A 3
8
Volume of B

5. Frustums of a Pyramid and a Circular Cone


For a frustum of a pyramid:
(a) Volume 10 cm
6 cm 8 cm
30 cm
= volume of larger pyramid - 24 cm
volume of smaller pyramid
(b) Total surface area of a frustum of a 18 cm

pyramid
= area of the upper base + frustum of a right circular cone
area of the lower base + Volume
total area of all lateral faces
= a 1 # r # 182 # 24 - 1 # r # 62 # 8k cm3
3 3
For a frustum of a circular cone: 3
= 2 496r cm
(a) Volume
Curved surface area
= volume of larger circular cone -
= (r # 18 # 30 - r # 6 # 10) cm2
volume of smaller circular cone
= 480r cm2
(b) Total surface area of a frustum of a
Total surface area
circular cone
= (r # 62 + r # 182 + 480r) cm2
= area of the upper base +
= 840r cm2
area of the lower base +
curved surface area
10.68 Chapter 10

✔ Checkpoint
In each of the following, put ‘✓’ in if it is correct, and ‘✗’ if it is wrong.

The figure shows a triangular pyramid VABC. [Nos. 1–2]


1. B is the projection of A on the plane VBC. V

2. VC is the corresponding height of the base VAB of the pyramid.


A B

C
The figure shows a regular pyramid. [Nos. 3–4]
3. All the lateral faces of the pyramid are equilateral triangles. 10 cm

4. The volume of the pyramid is 1 000 cm3.


3
10 cm

10 cm

5. In the figure, the total surface area of the solid right circular 18 cm

cone is 135r cm2.


15 cm

6. The volume of a hemisphere of radius 6 cm is 288r cm3.

7. The surface area of a sphere of diameter d m is rd2 m2.

8. In the figure, area of PQUT : area of PQRS = 4 : 9. 2 cm


P Q
3 cm
T U

S R
PTS and QUR are straight lines.

In the figure, a right circular cone P is divided into


Q h
a small circular cone Q similar to P and a frustum R.
[Nos. 9–10]
3h
P R 2h
9. Volume of Q : volume of R = 1 : 8
10. Curved surface area of P : curved surface area of Q
=9:1
Mensuration (III) 10.69

Supplementary Exercise 10
[In this exercise, unless otherwise stated, give the answers correct to
3 significant figures if necessary.]

Level 1

For each solid in Nos. 1–4, find


(a) the volume, (b) the total surface area.

1. 2.
12 cm 15 cm
24 m 25 m

9 cm 7m
right pyramid with a square base right circular cone

3. 4.

14 m

8 mm

sphere hemisphere

For each frustum of a right pyramid or a right circular cone in Nos. 5–6,
find
(a) the volume, (b) the total surface area.

5. 6.
10 cm 34 cm
30 cm
8 cm 16 cm
10 cm

8 cm 34 cm
30 cm

32 cm
24 cm 12 cm
The upper base and the
lower base are squares.

7. In the figure, ABCDEFGH is a cube of side 6 cm. Find the volume of E H

pyramid BFHG.
F G

D C

A 6 cm B
10.70 Chapter 10

8. The volume of a pyramid is 510 cm3 and its base is a right-angled


triangle whose legs are 20 cm and 9 cm long. Find the height of the
pyramid.

9. The figure shows a right pyramid. Its base is a square of side 10 cm


and its slant edge is 13 cm long. Find its total surface area. 13 cm

10 cm
10 cm

10. The figure shows a sphere. The area of a cross section which passes area of cross
section
through the centre is 144r cm2. Find the volume of the sphere in = 144r cm2
terms of r.

11. The frustum shown in the figure is formed by cutting a small right V
E
pyramid VEFGH from a large right pyramid VABCD. The bases of H
both pyramids are squares. Find the volume of the frustum. F G
5 cm 18 cm
12 cm
D C

A 15 cm B

3
12. The volume of a right circular cone is 578r cm and the
circumference of its base is 17r cm. Find the height of the circular
cone.

13. Find the total surface area of the right circular cone in the figure.
(Give the answer in terms of r.) 20 cm

21 cm

14. The figure shows a frustum of a right circular cone. The radii of the V

upper base and the lower base are 4 cm and 7 cm respectively. It is 6 cm 4 cm

given that VQ = 6 cm and QP = 4.5 cm. Find C Q D


4.5 cm
(a) the curved surface area of the frustum,
A P B
7 cm
(b) the total surface area of the frustum.

2
15. In the figure, the curved surface area of a hemisphere is 450r cm . curved surface area
= 450r cm2
Find its
(a) radius,

(b) volume in terms of r.


Mensuration (III) 10.71

16. The box in the figure contains five ping-pong balls each
with a radius of 2 cm. The box is a right prism of base area ping-pong balls
base area
= 18 cm2
18 cm 2 and length 21 cm. Find the volume of the 21 cm

remaining space in the box.

17. The solid in the figure is composed of a right circular cone and a
(r + 4) cm
hemisphere with a common base. If the curved surface area of the
cone is the same as that of the hemisphere, find the value of r.
r cm

18. Find the unknown in each of the following pairs of similar plane
figures, where the marked lengths are corresponding lengths.
(a) (b)
5 cm x cm
3 cm
2 cm

area = 30 cm
2
area = A cm
2
area = 64 cm2 area = 36 cm
2

19. Find the unknown in each of the following pairs of similar solids,
where the marked lengths are corresponding lengths.
(a) (b)

4 cm a cm
12 cm

8 cm

curved surface curved surface


2
volume = 324 cm3 volume = b cm3 area = 144 cm2 area = 36 cm

20. The figure shows a right pyramid whose base is a rhombus. The
diagonals of the base are 5 cm and 16 cm long. If the longer slant
17 cm
edge is 17 cm long, find the volume of the pyramid. m

16
5c

cm

21. Fig. A is a right cylinder of base radius 4 cm and height 14 cm. 4 cm

Fig. B is a sphere of radius r cm. The total surface area of the


cylinder is the same as the surface area of the sphere.
14 cm
(a) Find the value of r.
r cm
Explain (b) Is the volume of the cylinder the same as that of the sphere?
Explain your answer. Fig. A Fig. B
10.72 Chapter 10

22. Refer to the figure. The ice cream cone used by a shop is a right 3 cm
circular cone of base radius 3 cm and depth 15 cm. Bonnie buys two
ice cream spheres held by an ice cream cone from the shop. The
diameter of each ice cream sphere is 5 cm.
15 cm
(a) Find the total volume of the two ice cream spheres in terms of
r.
Explain (b) If the ice cream completely melts into the ice cream cone, will
the ice cream overflow? Explain your answer.
(Assume that there is no change in the volume of the ice cream
when it melts.)

23. In the figure, a right circular cone A is divided into a frustum C and
B h
a small circular cone B similar to A. Find the following ratios.
(a) Volume of B : volume of A 3h
A C 2h
(b) Volume of C : volume of A

(c) Curved surface area of B : curved surface area of C

Level 2
3 Q
24. Fig. A is a right circular cone of volume 1 500r cm
and height 20 cm. If the circular cone is cut along VQ, V Q
a sector shown in Fig. B is obtained. V
20 cm i
(a) Find the base radius of the circular cone.
P O Q
P
(b) Find the curved surface area of the circular cone
Fig. A Fig. B
in terms of r.
(c) Find i.

25. The regular tetrahedron in the figure is a pyramid in which the four
faces are congruent equilateral triangles. If the height of the regular
tetrahedron is 8 cm and its volume is 2 6 cm3, find its total
surface area.

26. Fig. A is a glass paperweight in the shape of a right pyramid. 6 cm

Its base is a square of side 8 cm and its height is 6 cm. Fig. B


3 cm
is a right cylindrical glass paperweight of height 3 cm and base 8 cm

diameter 8 cm. Which paperweight has a larger total surface 8 cm 8 cm

area? By how much? Fig. A Fig. B


Mensuration (III) 10.73

27. The figure shows a right pyramid whose base ABCD is a


26 cm
16 cm by 9 cm rectangle and its height is 26 cm. The
pyramid rests vertically in a tray and the depth of water in
the tray is 6.5 cm. Find the volume of the part of the pyramid 6.5 cm

below the water surface.


C

cm
D

9
A 16 cm B

6.5 cm

28. The figure shows an inverted right conical vessel standing O 7 cm


A
vertically and containing water to a depth of 12 cm.
(a) Find the volume of the water in the vessel in terms of r. C

16.8 cm
(b) If all the water in the vessel is poured into an empty
12 cm
cylindrical cup of base radius 4 cm and height 12 cm, find
the depth of water inside the cup.

29. Fig. A is a storage tank. The tank is composed of a right A


4m
cylinder of base radius 3.5 m and a right circular cone
B C
of slant height 4 m as shown in Fig. B. It is known that
the height of the cylinder is 5 times that of the cone.
Find
(a) the height of the cone and distance AX, Y X
3.5 m
(b) the total curved surface area of the storage tank. Fig. A Fig. B

12
30. The volume of the Earth is 1.1 # 10 km3. Jupiter and
Saturn are the two largest planets in the solar system.
Assume that the two planets are spheres. Use the
following information to estimate how many times the
volume of each planet is that of the Earth, correct to the
nearest 100 times. Jupiter Saturn

10
(a) The surface area of Jupiter is 6.4 # 10 km2.
10
(b) The surface area of Saturn is 4.2 # 10 km2.
10.74 Chapter 10

31. In the figure, the upper part of a glass test tube is a right cylinder of
height 16 cm and the lower part is a hemisphere of internal radius 2 cm.
16 cm
(a) Find the capacity of the test tube.

(b) Find the volume of the glass needed to make the test tube if the 2 cm
thickness of the tube is 1 mm.

32. In the figure, ADFB and AEGC are straight lines, and DE # FG # BC. A

If AD : DF : FB = 2 : 3 : 1, find the area of DEGF : the area of


FGCB. D E

F G
B C

33. In the figure, ABCD is a trapezium, where AB # DC. E is a point on A B

DC such that AE # BC. If AE and DB intersect at F and F


DF : FB = 2 : 1, find the area of 3ADF : the area of EFBC.

D C
E

34. A drink is sold in two different sizes. The cans used are similar in
shape. It is given that the volume of a small can is 320 cm3 and its
total surface area is 800 cm2. If the height of a large can is 25%
more than that of a small can, find
(a) the volume of a large can,

(b) the total surface area of a large can.

35. In the figure, a regular pyramid C is divided into a small pyramid X


similar to C and a frustum Y. Find
X
total area of all lateral faces of X
(a) , 8 cm 8 cm
total area of all lateral faces of Y
(b) total area of all lateral faces of C, C

(c) total surface area of Y.


Y

12 cm

36. In the figure, there is some water in an inverted right conical vessel.
The area of the wet surfaces of the vessel is 91 of that of the curved
surface inside the vessel. If the capacity of the vessel is 540 cm3,
find the volume of the water in the vessel.
Mensuration (III) 10.75

37. The figure shows an inverted right conical vessel which is placed
Explain
vertically. It contains 40 cm3 of water and the depth of water is 6 cm.
Jeremy claims that if 100 cm3 of water is poured into the vessel, the
9 cm
water will overflow. Do you agree? Explain your answer. 6 cm

38. It is known that prism P and prism Q in Fig. A are similar. Their
bases are squares and the ratio of their total surface areas is 49 : 81.

Q Q
P P

Fig. A Fig. B
3
Explain (a) If a sphere of volume 7 cm is attached to the centre of the
top of P and another sphere of volume 9 cm3 is attached to the
centre of the top of Q, as shown in Fig. B, are the two new
solids formed similar? Explain your answer.
Open-
ended
(b) If two cubes X and Y are to be attached to the top of prisms P
and Q respectively, such that the two new solids formed are
similar, suggest the lengths of sides of X and Y.
V
39.

15 cm

D C
10 cm

16 cm X
16 cm A 8 cm B

The figure shows a rectangular container filled up with water. Its


height is 10 cm and its base is a square of side 16 cm. A regular
pyramid VABCD with a height of 15 cm and a square base of side
16 cm is put into the container and water overflows. Find
(a) the volume of the water that overflows,

(b) (i) the length of VX,


(ii) the total area of all lateral faces of the pyramid,

(c) the total area of the wet parts on the lateral faces of the
pyramid.
10.76 Chapter 10

40. In the figure, a solid right circular cone of base radius r cm A

and height 20 cm is put into a rectangular container fully filled 10 cm

with water. The volume of water that overflows is 1 312.5r cm3 Y N 20 cm

and the height of the small pyramid above the water surface is
X r cm C
10 cm.
(a) Express the length of YN in terms of r.

(b) Find the value of r.

(c) Find the lengths of AC and AN.

(d) Find the total area of the wet surfaces of the circular cone in terms of r.

Level 3

41. (a) Fig. A shows a regular hexagon of side 6 cm. Find its area. A

(b) Fig. B shows a frustum of a right pyramid. The


upper base and lower base of the frustum are regular
M B
hexagons of sides 3 cm and 6 cm respectively.
3 cm
(i) Find AM : AN.
3
(ii) If the volume of the frustum is 427 cm , find the
6 cm N C
volume of the original pyramid.
(iii) Find the height of the frustum. 6 cm
Fig. A Fig. B

42. A sandglass is composed of two identical right circular cones, each


of height 12 cm. They are joined at their common vertex V. Fig. A 12 cm
and Fig. B show the sections of the sandglass at the beginning and 8 cm
V V
at the end respectively. At the beginning, the depth of sand is 8 cm.
It takes 4 minutes 16 seconds for all the sand to flow to the lower 12 cm

cone. If the depth of sand at the upper part increases to 10 cm at the


beginning, and the flowing speed remains unchanged, how long does Fig. A Fig. B
it take for all the sand to flow to the lower cone?

43. As shown in the figure, a sphere of radius r cm is put into an O 6 cm


B A
inverted right circular conical vessel filled up with water. O is
the centre of the circular base of the cone. G is the centre of the
G
sphere. AOB is a base diameter of the cone. OGH is a diameter of r cm
cm

the sphere. It is given that the base radius of the cone is 6 cm and Q P
10

its slant height is 10 cm.


V
(a) Prove that 3VOA a 3VPG.

(b) Find the value of r.

(c) Express the volume of water in the vessel in terms of r.

(d) If the sphere is taken out, find the depth of water in the vessel.
Mensuration (III) 10.77

Cross-topic Questions

44. It is known that 30% of the surface of the Earth is land.

(a) The radius of the Earth is 6 400 km. Find the total area of the
land on the Earth.
(b) The scale of the world map used by Mr Chan, a geography
teacher, is 1 : 20 000 000.
2
(i) Find the total area of the land on the map in cm .
2
(ii) If the area of China on the map is 240 cm , find the actual area
of China in km2.

45. Sandy measured the diameter of a hemisphere as 18 cm, correct to


the nearest cm. Find the minimum possible total surface area of the
hemisphere.

46. The figure shows a balloon (considered as a sphere) used in


meteorology. Its surface area is 2 800 cm2.
(a) Find its volume.

(b) Suppose this balloon has a leak and its surface area decreases
by 15%, but it is still in the shape of a sphere. Find
(i) its volume at this moment,
(ii) the percentage decrease in its volume.

47. The figure shows a right pyramid, where the length of each slant V

edge is 26 cm. Its base is a regular hexagon of side 10 cm and it can


be divided into 6 identical equilateral triangles.
26 cm
(a) Find the height VM of the pyramid.

(b) Find the length of MN. F E


(Leave the radical sign ‘ ’ in the answer.) A D
M
N
(c) Find the volume of the pyramid. B 10 cm C

(d) Find the total surface area of the pyramid.


10.78 Chapter 10

Multiple-choice Questions

48. Find the total surface area of the regular 52. In 3ABC, DE # BC and AC : EC = 3 : 1. If
pyramid in the figure. the area of BCED is 125 cm2, find the area
A. 260 cm2 12 cm
of 3ADE.
B. 360 cm2 A. 25 cm2 A

C. 400 cm2 B. 100 cm2


D. 620 cm2 10 cm C. 125 cm2 D
E
10 cm
D. 225 cm2 B
C
49. The figure shows a solid consisting of two
right circular cones with a common base. 53. The ratio of the total surface areas of two
Find the total surface area of the solid. similar solids is 16 : 81. If the volume of the
A. 33.6r cm 2 small solid is 128 cm3, then the volume of
4 cm
B. 34.8r cm2 3 cm the large solid is
C. 36.6r cm2 A. 288 cm3.
D. 38.4r cm2 B. 648 cm3.
7.2 cm C. 729 cm3.
D. 1 458 cm3.

50. Two solid metal spheres of radius 3 cm each 54. The upper base and the lower base of the
are melted and recast into a solid circular frustum of a right pyramid in the figure are
cone of base area 30r cm2. Find the height rectangles. Find the volume of the frustum.
of the circular cone. A. 1 190 cm3 B 30 cm A
21 cm
A. 1.2 cm B. 1 260 cm3 D
C
B. 3.6 cm C. 1 820 cm3 6 cm

C. 6 cm D. 1 890 cm3 7 cm G
F
H E
D. 7.2 cm 10 cm
V
51. The figure shows a hemisphere and a
55. In the figure, the circumferences of the
right cylinder with equal base radii. Their
upper base and the lower base of the frustum
curved surface areas are a cm2 and b cm2
of a right circular cone are 20r cm and
respectively. Which of the following is true?
30r cm respectively. If the curved surface
area of the frustum is 325r cm2, find the
3 cm 2 cm curved surface area of the circular cone
3 cm
VCD.
A. a = 2b A. 260r cm2 V

B. a =b B. 360r cm2
C. b 1a C. 585r cm2 C D

D. a 1b D. 810r cm2

A B
Mensuration (III) 10.79

Application of Maths
The Mathematics of A4 Paper
Among all different sizes of paper, A4 paper is the most commonly
used. Its dimensions are 29.7 cm by 21 cm. The origin of these lengths
involves the knowledge of mathematics.

By folding and cutting a piece of A4 paper in half, 2 pieces of A5


paper can be obtained as shown in the figure. The ratios of the
length to the width of the two kinds of paper are the same. Thus, the
shape of the figure remains the same in enlargement or reduction ✂
during copying. Also, the spaces in the paper can be fully utilized.
a A4 paper is similar to A5 paper.
2 A4 Paper A5 Paper
=c m
Area of A4 paper length of A4 paper
`
Area of A5 paper length of A5 paper
2
2=c m
length of A4 paper ◀ Length of A5 paper
width of A4 paper = width of A4 paper
Length of A4 paper
= 2
Width of A4 paper
i.e. The ratio of the length to the width of A4 paper is 2 : 1. A8
A6
A7 A4
In fact, the ratio of the length to the width of all A series paper is 2 : 1. A5
A2
After defining the area of A0 paper as 1 m 2 , we can calculate
A3
that the length and the width of A0 paper are about 118.9 cm and
84.1 cm respectively. By folding and cutting a piece of A0 paper in A0
half, 2 pieces of A1 paper can be obtained. After folding and cutting
paper 3 more times, A4 paper can be obtained. A1

Q&A Alvin has a picture of A3 size. If he wants to print the picture on a piece of
A4 paper, what percentage of reduction should be set?
(Give the answer correct to 2 significant figures.)
M
E

T
S
10.80 Chapter 10

Exam Get - Set -


Expert
Tutor
Short Question Tutor

Example 1
Fig. A shows a solid right circular cone of base radius
60 cm and height 80 cm.
(a) Find the volume of the cone in terms of r.
(b) A bucket is fully filled with water. The bucket is in
the shape of a frustum of an inverted right circular
cone and its capacity is 324 000r cm3. The circular
cone is now held vertically in the bucket as shown
in Fig. B. The height of the small circular cone Fig. A Fig. B

above the water surface is half of that of the large


circular cone. Cherrie claims that the volume of the
water remaining in the bucket is smaller than r m3.
4
Do you agree? Explain your answer.

(5 marks)

Solution
(a) Volume of the cone = 1 # r # 60 # 80 cm
2 3
3 1 for correct formula

= 96 000r cm3 1 for correct answer


3
(b) Let V cm be the volume of water that overflows. Diagram Clue
3
Volume of small cone height of small cone
=b l 1 for correct Consider Fig. B and the water that
Volume of large cone height of large cone overflows.
3 method
(96 000r - V) cm
= a 1 k3
96 000r cm
3 2
96 000r - V = 1
96 000r 8

96 000r - V = 1 # 96 000r
8
Volume of water volume of frustum
V = 84 000r that overflows
=
under water surface

Volume of the water remaining in the bucket


= capacity of bucket - volume of water that overflows
= (324 000r - 84 000r) cm3 1 for correct method
3
= 240 000r cm
= 0.24r m3
1 r m3 ◀ r = 0.25r
4 4
` The claim is agreed. 1 for correct conclusion with reasons
Mensuration (III) 10.81

Exam Drill 1
Fig. A shows a solid regular pyramid. Its base is a square
of side 12 cm and its height is 18 cm.
(a) Find the volume of the pyramid.
(b) A container in the shape of a prism of height 12 cm
is fully filled with water. The base of the container
is a square of side 20 cm. The pyramid is now
held in the container as shown in Fig. B. Timothy Fig. A Fig. B
claims that the volume of the water remaining in the
container is greater than 4 000 cm3. Do you agree?
Explain your answer.
(5 marks)

Solution
10.82 Chapter 10

Public Exam Questions

HKDSE 2019 (A1)

1. The sum of the volumes of two spheres is 324r cm3. The radius of the
larger sphere is equal to the diameter of the smaller sphere. Express,
in terms of r,
(a) the volume of the larger sphere;

(b) the sum of the surface areas of the two spheres.

HKDSE 2018 (A2)

2. A right circular cylindrical container of base radius 8 cm and height


64 cm and an inverted right circular conical vessel of base radius
20 cm and height 60 cm are held vertically. The container is fully
filled with water. The water in the container is now poured into the
vessel.
(a) Find the volume of water in the vessel in terms of r.

(b) Find the depth of water in the vessel.

(c) If a solid metal sphere of radius 14 cm is then put into the


vessel and the sphere is totally immersed in the water, will the
water overflow? Explain your answer.
HKDSE 2017 (A2)

3. A solid metal right prism of base area 84 cm2 and height 20 cm is


melted and recast into two similar solid right pyramids. The bases of
the two pyramids are squares. The ratio of the base area of the
smaller pyramid to the base area of the larger pyramid is 4 : 9 .
(a) Find the volume of the larger pyramid.

(b) If the height of the larger pyramid is 12 cm, find the total
surface area of the smaller pyramid.
HKDSE 2016 (A2)

4. An inverted right circular conical vessel contains some milk. The vessel
is held vertically. The depth of milk in the vessel is 12 cm. Peter then
pours 444r cm3 of milk into the vessel without overflowing. He now
finds that the depth of milk in the vessel is 16 cm.
(a) Express the final volume of milk in the vessel in terms of r.

(b) Peter claims that the final area of the wet curved surface of the
vessel is at least 800 cm2. Do you agree? Explain your answer.
Mensuration (III) 10.83

HKDSE 2014 (A2)

5. The figure shows a vessel in the form of a frustum which 72 cm


is made by cutting off the lower part of an inverted right

60 cm
28 cm
circular cone of base radius 72 cm and height 96 cm. The

96 cm
height of the vessel is 60 cm. The vessel is placed on a
horizontal table. Some water is now poured into the vessel.
John finds that the depth of water in the vessel is 28 cm.
(a) Find the area of the wet curved surface of the vessel
in terms of r.
(b) John claims that the volume of water in the vessel
is greater than 0.1 m3. Do you agree? Explain your
answer.

HKDSE 2013 (A2)

6. In a workshop, 2 identical solid metal right circular cylinders of base


radius R cm are melted and recast into 27 smaller identical solid
right circular cylinders of base radius r cm and height 10 cm. It is
given that the base area of a larger circular cylinder is 9 times that
of a smaller one.
(a) Find
(i) r : R,
(ii) the height of a larger circular cylinder.

(b) A craftsman claims that a smaller circular cylinder and a


larger circular cylinder are similar. Do you agree? Explain your
answer.

HKDSE 2012 (A2)

7. Fig. A shows a solid metal right circular cone of base radius


48 cm and height 96 cm.
(a) Find the volume of the circular cone in terms of r.
96 cm

(b) A hemispherical vessel of radius 60 cm is held


vertically on a horizontal surface. The vessel is fully
48 cm
filled with milk.
Fig. A Fig. B
(i) Find the volume of the milk in the vessel in terms of r.
(ii) The circular cone is now held vertically in the vessel
as shown in Fig. B. A craftsman claims that the
volume of the milk remaining in the vessel is
greater than 0.3 m3. Do you agree? Explain your
answer.
10.84 Chapter 10

HKDSE 2019 (A) HKDSE 2017 (A)

8. The base of a solid right pyramid is a square 12. I n t h e f i g u r e , A B C D a n d B E D F a r e


of side 18 cm. If the height of the pyramid parallelograms. E is a point lying on BC
is 12 cm, then the total surface area of the such that BE : EC = 2 : 3. AC cuts BF and
pyramid is DE at G and H respectively. If the area of
A. 432 cm2. 3ABG is 135 cm 2 , then the area of the
B. 540 cm2. quadrilateral DFGH is
F
C. 756 cm2. A. 60 cm2. D A

D. 864 cm2. B. 81 cm2. G


H
C. 90 cm2.
HKDSE 2019 (A) D. 144 cm2.
C B
E
9. In the figure, ABCD is a parallelogram and
AEFG is a square. It is given that HKDSE 2016 (A)

BE : EF : FC = 2 : 7 : 3. BD cuts AE and 13. In the figure, ABCD, CDEF and EFGH


FG at the points X and Y respectively. If the are squares. AG cuts CD and EF at P
area of 3ABX is 24 cm2, then the area of and Q respectively. Find the ratio of the
the quadrilateral CDYF is area of quadrilateral DEQP to the area of
A. 54 cm2. C F E B
quadrilateral ABCP.
B. 77 cm2. X
A. 1:2 A D E H

C. 81 cm2. Y
B. 2:3
P
D. 87 cm2. C. 3:5 Q
D G A
D. 4:9 B G
C F
HKDSE 2017 (A)

10. The scale of a map is 1 : 20 000. If the area HKDSE 2015 (A)

of a zoo on the map is 4 cm2, then the actual 14. T h e h e i g h t a n d t h e b a s e r a d i u s o f a


area of the zoo is right circular cone are 12 cm and 9 cm
4 2 respectively. The figure shows a frustum
A. 8 # 10 m .
B. 1.6 # 105 m2. which is made by cutting off the upper
C. 3.2 # 105 m2. part of the circular cone. The height of the
D. 1 # 106 m2. frustum is 8 cm. Find the volume of the
frustum.
HKDSE 2017 (A) A. 210r cm3
11. The base radius of a right circular cone is B. 312r cm3
2 times the base radius of a right circular C. 324r cm3
cylinder while the height of the circular D. 936r cm3
cylinder is 3 times the height of the circular
cone. If the volume of the circular cone is
36r cm 3, then the volume of the circular
cylinder is
3 3
A. 27r cm . C. 81r cm .
3 3
B. 48r cm . D. 144r cm .
Mensuration (III) 10.85

HKDSE 2015 (A) HKDSE 2013 (A)

15. In the figure, ABCD is a parallelogram. E is 18. In the figure, the solid consists of a right
a point lying on CD such that circular cone and a hemisphere with a
DE : EC = 2 : 3. AD produced and BE common base. The base radius and the
produced meet at F while AE produced height of the circular cone are 3 cm and
and BC produced meet at G. If the area of 4 cm respectively. Find the total surface
3DEF is 8 cm2, then the area of 3CEG is area of the solid.
A. 12 cm2. A D F A. 30r cm2
B. 18 cm2. B. 33r cm2
E
C. 20 cm2. C. 48r cm2
D. 27 cm2. D. 51r cm2
G
B C

HKDSE 2014 (A)

16. In the figure, B is a point lying on AC such


HKDSE 2012 (A)
that AB : BC = 3 : 2. It is given that
19. In the figure, ABCD is a parallelogram.
AE # BD. If the area of 3BCD and
E and F are points lying on AB and CD
the area of 3CDE are 4 cm2 and 8 cm2
respectively. AD produced and EF produced
respectively, then the area of the trapezium
meet at G. It is given that DF : FC = 3 : 4
ABDE is
and AD : DG = 1 : 1. If the area of 3DFG
A. 18 cm2. E
is 3 cm2, then the area of the parallelogram
B. 21 cm2.
ABCD is
C. 27 cm2. G
D A. 12 cm2.
D. 33 cm2.
A C B. 14 cm2.
B
C. 18 cm2. D F C
HKDSE 2013 (A)
2
D. 21 cm2.
17. The actual area of a playground is 900 m .
If the area of the playground on a map is A E B

36 cm2, then the scale of the map is


A. 1 : 25.
B. 1 : 50.
C. 1 : 500.
D. 1 : 250 000.
10.86 Chapter 10

Answers
Q & A (P. 10.1) Class Practice 10.1 (P. 10.12)
2 2 3 3 3 2
(a) small circle: r cm , large circle: 9r cm 1. 210 m 2. 50 cm 3. 400 cm 4. 132 m
2 2
(b) 9 5. 384 cm 6. 736 cm 7. 12 cm
2
Warm-up Exercise (P. 10.2) 8. (a) 24 cm (b) 6 cm (c) 10 cm
2
1. circumference = 12r m, area = 36r m Exercise 10A (P. 10.13)
2 3 3 3
2. (a) 10r mm (b) 75r mm 1. 88 mm 2. 200 m 3. 154 m
3 2 2 2 2
3. (a) 240 cm (b) 300 cm 4. 209 mm 5. 765 cm 6. 168 cm
3 2 2
4. (a) 28r cm (b) 28r cm (c) 36r cm 7. 7 8. 6 9. 8 10. 138 m
2
5. 7.5 11. (a) 720 cm (b) 18 cm
Instant Drill 12. (a) 20 (b) 25.6 cm
3
1 9 cm 13. (a) 8 cm (b) 360 cm
3 2
2 (a) 7.2 cm (b) 60 cm 14. (a) 24 cm (b) 1 360 cm
2 3
3 85 cm 15. (a) 12 cm (b) 5 cm (c) 160 cm
2 2
4 (a) 43.3 cm (b) 173 cm 16. no
3 3
5 768r cm 17. (a) 15 cm (b) 20 cm (c) 2 880 cm
2
6 9 cm 18. (a) 24 cm (b) 6 cm
2
7 17 cm 19. 768 cm
3 2
8 (a) 6 cm (b) 96r cm 20. (a) VM = 10 cm, VN = 17 cm (b) 864 cm
3 2 2
9 45r m 21. (a) 60 cm (b) VK = 12.4 cm, area = 49.5 cm
2
10 3.4 cm (c) 299 cm
2 3
11 6.6 cm 22. (a) 80 cm (b) 6 cm (c) 512 cm
2 2 2
12 (a) 5 cm (b) 39r cm 23. (a) 48 cm (b) 26 cm (c) 152 cm
2 2
13 63 m 24. (a) 3 3 cm (b) 36 3 cm
3
14 6 000 (c) 2 6 cm (d) 18 2 cm
2 2 2
15 (a) 3 cm (b) 27 cm (c) 21 cm Quick Quiz (P. 10.18)
2 3
16 12.5 cm 60r cm
3
17 (a) 5 : 6 (b) 86.4 m Quick Quiz (P. 10.21)
3 2
18 525 cm 14r cm
3
19 (a) 3 cm (b) 252r cm Class Practice 10.2 (P. 10.23)
3
20 (a) 6 cm (b) 304 cm 1. 737 m
3
2. 1 020 cm
3
3. 302 cm
2
4. 283 mm
2

21 117 : 8 5. (a) 24 cm (b) 704 cm


2

2
22 2 052 cm 6. (a) 29 cm (b) 9 240 cm
3

23 (a) VH = 3 cm, VD = 10 cm (b) $44.0 Exercise 10B (P. 10.24)


2
24 451.84r cm 1. 196r cm
3
2. 1 014r cm
3
3. 320r cm
3

Quick Quiz (P. 10.4) 4. (a) 8 m (b) 4 cm


S, Q 5. (a) 320r cm
2
(b) 576r cm
2

Quick Quiz (P. 10.7) 6. (a) 476r cm


2
(b) 672r cm
2

3 2 2
640 cm 7. (a) 260r cm (b) 360r cm
Quick Quiz (P. 10.9) 8. (a) (i) 5 mm (ii) 12 cm
2
216 cm (b) (i) 9m (ii) 15 cm
9. yes
Mensuration (III) 10.87

3
10. (a) 3.2 m (b) 19.3 m 25. 3.75 cm
2
11. (a) 25 cm (b) 550 cm 26. (a) 5 cm (b) 25.6%
2
12. (a) 8 cm (b) 17 cm (c) 200r cm 27. (a) 1 : 4
2
13. (a) 12 cm (b) 15 cm (c) 216r cm (b) (i) r = 3, R = 12 (ii) no
3
14. (a) 10 cm (b) 24 cm (c) 800r cm Quick Quiz (P. 10.39)

15. 4.86 cm 25
36
3 2
16. (a) 67.2r m (b) 45.6r m Quick Quiz (P. 10.43)
3 2 9
17. (a) 126 cm (b) (i) 107 cm (ii) 169c (a) (b) 27
16 64
18. (a) base radius = 7.01 cm, height = 24.0 cm
Class Practice 10.4 (P. 10.45)
3
(b) 1 240 cm
1. 4 : 49
2 3
19. (a) 16.8 cm (b) 823.2r cm (c) 3 292.8r cm 2
2. 18 cm 3. 12 mm
3
20. 94.2 cm
4. (a) 9 : 16 (b) 27 : 64
3
21. (a) 256r cm (b) 12 cm 2 3
5. (a) 200 cm (b) 32 cm
22. (a) 17 cm
Exercise 10D (P. 10.46)
(b) base radius = 16 cm, height = 5.74 cm
1. 1 : 4 2. 9 : 49 3. 72
(c) A and B
4. 40.5 5. 20 6. 12
Quick Quiz (P. 10.29)
7. (a) 25 : 9 (b) 125 : 27
524 m3
8. (a) 4 : 81 (b) 8 : 729
Quick Quiz (P. 10.31)
9. 28 10. 768 11. 7
64r m2 2
12. 3 13. 225.6 cm
Class Practice 10.3 (P. 10.33)
14. (a) 3 : 4 (b) 32 cm
2
(c) 14 cm
2
3 2
1. (a) volume = 2 140 cm , surface area = 804 cm 15. (a) 3 : 1 (b) 9 : 1
3 2
(b) volume = 2 090 cm , total surface area = 942 cm 16. (a) 36 : 25 (b) 6 : 5 (c) 216 : 125
3
2. (a) 5 mm (b) 524 mm 17. 8
3 2
3. (a) 240r m (b) 132r m 18. (a) 1 : 3 (b) 1 : 8
Exercise 10C (P. 10.34) 19. (a) 8 cm
2
(b) 8 cm
2

3 2
1. (a) 972r cm (b) 324r cm 20. (a) 2 : 3 (b) 0.8r m
2

3 2
2. (a) 0.288r m (b) 1.44r m 21. (a) 80 cm
2
(b) 135 cm
3

3 2
3. (a) 2 250r cm (b) 675r cm 22. (a) 126 cm
2
(b) 135 cm
3

4. 4 5. 2 6. 20 23. (a) 30% (b) 119.7%


3
7. 14 8. 6 m 9. 2 090 cm 24. 1 : 7
2 3 3
10. 1 890 m 11. 486r cm 12. 890 m 25. (a) 2 : 1 (b) 40 mm (c) no
2
13. (a) 13 cm (b) 115r cm 26. 32 cm
2

3
14. (a) 6 cm (b) 288r cm 27. (a) 900 cm
3
(b) (i) 100 cm
3
(ii) no
2
15. (a) 4 cm (b) 64r cm Class Practice 10.5 (P. 10.61)
2 2 2
16. (a) 18r cm (b) 50r cm (c) 84r cm 1. 380 cm
3
2. 895 cm
3

17. 16 cm 3. (a) 534 cm


2
(b) 1 190 cm
2
3
3
18. (a) 2 520 mm (b) 75.6 g
Exercise 10E (P. 10.62)
2
19. 0.32 cm 20. 532r cm 21. 616 kg 3 3 3 3
1. 370 m 2. 1 176 cm 3. 198 cm 4. 205 cm
2 3
22. (a) 15 cm (b) 825r cm (c) 3 750r cm 2 2
5. (a) 4 500 cm (b) 9 000 cm
23. (a) 3 cm (b) no
3 3
24. (a) 843 cm (b) 843 cm
10.88 Chapter 10

2 2 3 3
6. (a) 408r cm (b) 728r cm 31. (a) 218 cm (b) 23.2 cm
2
7. (a) 320r cm (b) 30 cm 32. 21 : 11 33. 2 : 5
2 2 3 2
(c) 720r cm (d) 400r cm 34. (a) 625 cm (b) 1 250 cm
4 (c) 1 024 cm
2 3 2 2
8. (a) 4 320 cm (b) 14 800 cm 35. (a) (b) 240 cm
5 3
3 3
9. (a) 5 cm (b) 390r cm 36. 20 cm 37. yes
10. (a) 10 cm (b) $1 090 38. (a) no
3
11. 16r cm (b) X: 7 cm, Y: 9 cm (or other reasonable answers)
3 3 2
12. (a) 10 cm (b) 1 400 cm 39. (a) 1 230 cm (b) (i) 17 cm (ii) 544 cm
3 2
13. (a) 15 (b) 3 325 cm (c) 484 cm
14. 35 cm
3
40. (a) r cm (b) 15
2
3 2
15. (a) 48 cm (b) 6 970 cm (c) 1 660 cm (c) AC = 25 cm, AN = 12.5 cm
(b) 21 1 cm
2 2
16. (a) (i) no (ii) no (d) 506.25r cm
3
2 2
17. (a) 447 cm (b) no 41. (a) 93.5 cm
3 3
18. (a) 270 cm (b) 2 cm (b) (i) 1:2 (ii) 488 cm (iii) 7.83 cm

Checkpoint (P. 10.68) 42. 8 minutes 20 seconds


3
1. ✓ 2. ✗ 3. ✓ 4. ✗ 43. (b) 3 (c) 60r cm (d) 6.84 cm
2
5. ✗ 6. ✗ 7. ✓ 8. ✗ 44. (a) 154 000 000 km

9. ✗ 10. ✓ (b) (i) 3 860 cm2 (ii) 9 600 000 km


2

2
Supplementary Exercise 10 (P. 10.69) 45. 722 cm
3 2 3
1. (a) 1 296 cm (b) 864 cm 46. (a) 13 900 cm

2. (a) 1 230 m
3
(b) 704 m
2 (b) (i) 10 900 cm3 (ii) 21.6%
3 2 47. (a) 24 cm (b) 5 3 cm
3. (a) 11 500 m (b) 2 460 m
3 2 3 2
4. (a) 1 070 mm (b) 603 mm (c) 2 080 cm (d) 1 030 cm
3 2 48. B 49. D 50. D 51. C
5. (a) 2 688 cm (b) 1 440 cm
3 2 52. B 53. D 54. C 55. A
6. (a) 56 300 cm (b) 9 150 cm
3 2
7. 36 cm 8. 17 cm 9. 340 cm Application of Maths
3 3
10. 2 304r cm 11. 1 300 cm 12. 24 cm 71%
2
13. 1 050r cm Exam Get-Set-Go (P. 10.80)
2 2
14. (a) 187 cm (b) 391 cm Exam Drill
3 3
15. (a) 15 cm (b) 2 250r cm 1 (a) 864 cm (b) no
3
16. 210 cm 17. 4
Public Exam Questions
18. (a) 4.8 (b) 4 3 2
1. (a) 288r cm (b) 180r cm
19. (a) 96 (b) 2 3
2. (a) 4 096r cm (b) 48 cm (c) no
3
20. 200 cm 3 2
3. (a) 1 296 cm (b) 384 cm
21. (a) 6 (b) no 3
4. (a) 768r cm (b) no
125
22. (a) r cm3 (b) no 2
3 5. (a) 2 625r cm (b) yes
23. (a) 1 : 27 (b) 26 : 27 (c) 1 : 8
6. (a) (i) 1:3 (ii) 15 cm (b) no
2
24. (a) 15 cm (b) 375r cm (c) 216c 3
7. (a) 73 728r cm
2 2
25. 20.8 cm 26. paperweight in Fig. A, 3.45 cm
(b) (i) 144 000r cm3 (ii) no
3
27. 721.5 cm
8. D 9. D 10. B 11. C
3
28. (a) 100r cm (b) 6.25 cm
12. D 13. C 14. B 15. D
2
29. (a) height: 1.94 m, AX: 12.1 m (b) 257 m
16. D 17. C 18. B 19. B
30. (a) 1 400 (b) 700

You might also like